Bar Prep Final Exam Spring 2021

¡Supera tus tareas y exámenes ahora con Quizwiz!

A homeowner said to a carpenter, "My porch is need of repair. I think the boards are cracked. Can you do the job for $5,000?" The carpenter replied, "I'd be happy to, but $5,000 seems a little low. Make it $6,000 and you got a deal." The homeowner then remarked, "Deal." Three days later, the carpenter purchased the necessary materials, then drove his pickup truck to the homeowner's home and unloaded the materials and equipment needed to perform the job. When the homeowner looked out his window and saw what was transpiring, he immediately ran outside and exclaimed, "Stop! The deal's off. I decided to repair the porch myself." In an action by the carpenter against the homeowner for breach of contract, which of the following is the most accurate? (A) A bilateral contract was formed before the carpenter purchased the materials. (B) A unilateral contract was formed when the carpenter started to perform. (C) No contract was formed, because the carpenter did not start to perform. (D) No contract was formed, because the carpenter rejected the homeowner's terms.

(A) A bilateral contract was formed before the carpenter purchased the materials. A bilateral offer, which seeks a return promise rather than the performance of the requested act as acceptance, becomes enforceable as soon as the offeree has promised to perform the act. Thus, this answer choice is correct. The homeowner and carpenter had a valid, enforceable agreement as soon as the two agreed to each other's terms.

A buyer ordered 1,000 widgets from a seller for immediate delivery. The seller responded by shipping 800 widgets rather than the full 1,000 ordered because the seller did not have an adequate inventory and thought it better to ship something rather than nothing. Which of the following is the most accurate statement? (A) A contract was formed when the seller shipped the widgets. The UCC governs contracts for the sale of goods. (B) A contract will be formed once the widgets arrive at the buyer's address. (C) No contract exists, because the seller shipped nonconforming goods. (D) No contract exists, because the seller did not cure the nonconforming tender immediately.

(A) A contract was formed when the seller shipped the widgets. The UCC governs contracts for the sale of goods. Under the UCC, an offer can be accepted by the offeree communicating acceptance or by the prompt shipment of the goods. When the seller ships nonconforming goods, this operates as an acceptance of the offer and a simultaneous breach of contract.

A heavyset man with long red hair robbed a liquor store. Thereafter, a man was arrested and charged with armed robbery. At the man's trial, the owner of the liquor store was called to testify. He admitted that he was unable to identify the man, who now had a shaven head, as the robber. The prosecuting attorney then handed the owner six photographs. He proposed to testify, over defense objections, that he had previously told the prosecuting attorney that picture #4, admittedly a picture of the man before he shaved his head, was a picture of the person who robbed his store. How should the court rule on the owner's proffered testimony? (A) Admissible, as a prior identification by the witness. (B) Admissible, as a past recollection recorded. (C) Inadmissible, as hearsay not within any recognized exception. (D) Inadmissible, because it is a violation of the man's right of confrontation.

(A) Admissible, as a prior identification by the witness. FRE 801(d)(1)(C) defines as nonhearsay a prior statement by a witness of identification of a person made after perceiving him, if the person making the identification is available to testify at the trial or hearing and subject to cross-examination concerning the statement. There is no requirement that the witness first be impeached, nor does a prior identification require that the identifying witness make a positive in-court identification. This choice is therefore correct, because the owner had previously identified the man in picture #4.

A defendant was on trial for attempted murder. The alleged victim was called by the prosecution to testify. During her testimony, the victim recounted the incident and described how the defendant savagely beat her with a baseball bat. She was not asked by the prosecution whether she made any statements during the attack. After the victim was excused and left the witness stand, the prosecution called another witness to testify. The witness proposes to testify that when the beating stopped, the victim screamed, "I'm dying. Don't let the defendant get away with this." Upon objection by the defendant's attorney, how will the court rule on the witness's proffered testimony? (A) Admissible, as an excited utterance. (B) Admissible, as a dying declaration. (C) Inadmissible, as hearsay not within any recognized exception. (D) Inadmissible, because the victim was not questioned about the statement before being excused.

(A) Admissible, as an excited utterance. Under FRE 803(2), an excited utterance is defined as a statement relating to a startling event or condition made while the declarant was under the stress of excitement caused by the event or condition. The witness's testimony of the victim's statement, "I'm dying. Don't let the defendant get away with this," related to the savage beating (the startling event) and was made when the beating stopped (while the victim was still under the stress of excitement). The testimony will be admissible substantively as an excited utterance.

After being passed over for a promotion, an aeronautic engineer became a whistleblower. He contacted a government agency to state that the company for which he worked was submitting false safety reports on newly built aircraft. When the company learned that the engineer was leaking this information to the government agency, he was fired from his job. Afterward, the engineer sued the company for wrongful termination of employment. During the discovery stage of litigation, the engineer was deposed by the company's attorney. In his deposition, the engineer stated that the company submitted false safety reports to the government agency to cover up structural defects in its aircraft. A pilot was injured when one of the company's airplanes he was piloting crashed. The pilot asserted a strict product liability tort against the company. At trial, the pilot sought to introduce into evidence portions of the engineer's deposition from his wrongful termination lawsuit against the company. Assume that the engineer is unavailable to testify at trial. Upon objection by the company, how should the trial court judge rule on the engineer's deposition? (A) Admissible, as former testimony. (B) Admissible, as a vicarious statement. (C) Inadmissible, as hearsay not within any recognized exception. (D) Inadmissible, because the company did not have the opportunity to cross-examine the engineer on the liability issue for which the statement is now being offered.

(A) Admissible, as former testimony. Usually, former testimony questions will involve the same parties where each had the opportunity to interrogate the witness at the earlier trial. In this question, we have a plaintiff (who was not party to the first suit) seeking to introduce former testimony against the same defendant. According to Lilly, "using the testimony against the same defendant, which was a party to both suits and had an opportunity at the first trial to interrogate the witness, appears to fit comfortably within the exception." Evidence, p. 286. On the other hand, Lilly points out that if a plaintiff seeks to use all or part of the former testimony against a new defendant (who did not have an opportunity to interrogate the witness), then the testimony should be excluded. Here, because the testimony is being used against the same defendant, it is admissible.

A plaintiff sued a defendant for injuries suffered in a fall on the sidewalk of the defendant's home. The plaintiff's complaint alleged that the walk was covered by a thick sheet of ice, which had been negligently left there for several days by the defendant. In his answer, the defendant set forth that the ice formed overnight and that the plaintiff fell before the defendant had a chance to remove it. During the trial, a physician, whose office was next door to the defendant's home, testified that he saw the plaintiff fall and that the ice, which had been there for several days was at least two inches thick. On cross-examination, counsel for the defendant asked the physician the following question: "During your treatment of the plaintiff on the day in question, is it not true that he told you his fall resulted from a loss of equilibrium after he suffered from dizziness?" Upon objection by the plaintiff's attorney, how should the court rule on the physician's testimony? (A) Admissible, because it is reasonably pertinent to diagnosis or treatment. (B) Admissible, because the physician had personal knowledge of the plaintiff's injuries. (C) Inadmissible, because the defendant's counsel failed to lay a proper foundation. (D) Inadmissible, because the physician's testimony is not relevant to prove that the plaintiff's alleged injuries are false or exaggerated.

(A) Admissible, because it is reasonably pertinent to diagnosis or treatment. The Federal Rules of Evidence allows not only statements of past symptoms and medical history as exceptions under the hearsay rule, but also the cause or source of a patient's past physical condition, insofar as it is reasonably pertinent to diagnosis or treatment. In this regard, the plaintiff's statements to the physician would be admissible as reasonably pertinent to diagnosis and/or treatment regarding the cause or source of his injuries.

A fan attended a minor league hockey game in his hometown. Unfortunately, he was only able to obtain tickets in the visitor's section. While at the game, he became involved in an altercation with a fan of the visiting team. When the fan cheered for a home team goal, the visiting fan turned around and threatened to kill the home fan if he didn't shut up. The home fan pulled a knife out of his pocket and stabbed the visiting fan in the arm. At his trial for aggravated assault, the home fan wants to introduce a statement from a witness who was standing next to the visiting fan at the game. The statement, which the witness had made earlier in the game when the home fan cheered for the home team, was, "You'd better watch out. At a hockey game last week, the visiting fan put two guys in the hospital when they wouldn't shut up. One of them had 33 stitches after the visiting fan bashed his head against the steps." Assume that the witness's statement is being offered as proof of the effect it produced in the home fan's mind. In this regard, how will the statement most likely be found? (A) Admissible, because the testimony is not hearsay as it is not being offered for its truth. (B) Admissible, as a present sense impression. (C) Inadmissible, as hearsay not within any recognized exception. (D) Inadmissible, because the statement is self-serving.

(A) Admissible, because the testimony is not hearsay as it is not being offered for its truth. This answer choice is correct, because the witness's statement that the visiting fan had put two men in the hospital at a hockey game the week before is being offered to show its effect on the defendant's state of mind, rather than being offered to prove the truth of the assertion. Thus, the witness's statement bears upon the reasonableness of the home fan's fear to justify his self-defense plea.

A man who had recently lost his job decided to seek revenge by committing a fake robbery at his former place of employment. The defendant entered the sporting goods store, picked up a ski mask and replica gun, and approached the clerk. The man pointed the gun at the clerk and demanded the money from the register. Terrified, the clerk closed the register and ran into the back room. The man panicked, ran back to his car, and drove off. As he was approaching a traffic light, he hit the gas and sped through the intersection, causing a truck to swerve out of the way and run over a pedestrian standing on the sidewalk. The pedestrian was rushed to the hospital, but died in surgery as a result of his injuries. This jurisdiction criminalizes speeding in an intersection. Which of the following is the most serious crime for which the man can be convicted? (A) Felony murder. (B) Involuntary manslaughter, because no robbery occurred. (C) Misdemeanor manslaughter, because speeding in an intersection is a crime. (D) Robbery, because the truck swerving into the pedestrian was a superseding cause.

(A) Felony murder. The pedestrian's death was unintentional, but did occur as a foreseeable result during the commission of the robbery (as the man had yet to reach a place of safety, terminating the felony). It is immaterial if the man's car actually hit the pedestrian or if the truck hit the pedestrian, because the man will still be held criminally responsible for the death.

The defendant and the victim have been rivals for years. Their rivalry began when they were teenagers and competed against each other on rival sports teams at schools on opposite sides of the town. Later, the victim broke up the defendant's marriage by having an affair with the defendant's wife. The defendant and the victim are now in rival motorcycle gangs, and each is trying to control the local drug trade. The defendant just learned that the victim has been dating his little sister, and she is now addicted to drugs. To make matters worse, the defendant also learned that after the victim gives the defendant's sister drugs, he passes her around to members of his gang so that they can "do whatever they want with her." The defendant is completely enraged by the victim's conduct. He calls the victim and asks if they can meet for a few beers to discuss a merger of their respective drug trade organizations. The victim agrees, so they meet and talk for an hour or so. Later, the defendant offers the victim a ride home. When they pull up in front of the victim's house, the defendant shoots the victim in the back of the head, stating, "That's for my little sister." The victim dies instantly. What is the most serious crime of which the defendant can be found guilty? (A) First-degree murder. (B) Second-degree murder. (C) Voluntary manslaughter. (D) Involuntary manslaughter.

(A) First-degree murder. First-degree murder includes intent-to-kill murder committed with premeditation and deliberation, felony murder, and in some jurisdictions, murder accomplished by lying in wait, poison, terrorism, or torture. If a murderer does any reflection or premeditation, even if the reflection is cursory and brief, he may be guilty of first-degree murder. An intentional killing may be mitigated to voluntary manslaughter (also known as a heat-of-passion killing) if the killing was mitigated by adequate provocation or other circumstances negating malice aforethought. The time period between the heat of passion and the fatal act must be short enough that a reasonable person would not have had time to cool off. Here, although the defendant was provoked by the victim's harmful acts to the defendant's sister, there was a significant cooling off period between the time the defendant learned of the victim's actions and the time he killed the victim. Moreover, the facts indicate that the murder was premeditated, as the defendant planned to take the victim out for drinks on the pretense of discussing business then drive him home and shoot him. Therefore, it is unlikely that the defendant's act will be mitigated to voluntary manslaughter, and the defendant will most likely be found guilty of first-degree murder.

A man took two people hostage and arranged for them to play a game. The man blindfold the first hostage and strapped a gun, which was pointed at the first hostage's head, to the second hostage's hand. The man told the second hostage that if he did not pull the trigger, that the man would shoot the second hostage. Not wanting to die, the second hostage pulled the trigger, killing the first hostage. If the second hostage is charged with the death of the first hostage, how should the jury find the second hostage? (A) Guilty of murder. (B) Guilty of manslaughter. (C) Not guilty, because the second hostage was under duress. (D) Not guilty, because the killing was done out of necessity

(A) Guilty of murder. The 2nd hostage intentionally pulled the trigger knowing that the gun was pointed at the first hostage's head. He chose to kill rather than to be killed, and he would therefore be guilty of murder. Note that duress is a defense to everything except murder, because one person's life is not more important than anyone else's.

A man recently moved to a new town and fell in love with its charm. A month later, deciding to get as involved as possible, the man submitted his name for an upcoming election for city council. However, the city refused to put his name on the ballot, citing a statute requiring residents to have lived within the town boundaries for at least 60 days prior to being considered for candidacy. The man sued the city, claiming that the statute violated the Due Process Clause. How should the court rule? (A) In favor of the city, because under a rational basis analysis, the statute is rationally related to a legitimate state interest. (B) In favor of the city, because under a strict scrutiny analysis, the statute is necessary to further a compelling state interest. (C) In favor of the man, because under a rational basis analysis, the 60-day residency requirement is unconstitutional. (D) In favor of the man, because under a strict scrutiny analysis, the 60-day residency requirement is unconstitutional.

(A) In favor of the city, because under a rational basis analysis, the statute is rationally related to a legitimate state interest. While the right to vote is typically considered a fundamental right subject to strict scrutiny, government regulations of ballot access by candidates based on age, duration of residency, or payment of filing fees require only a minimum rational basis scrutiny. Because the statute in question here is setting a durational residency requirement on a person's being able to get their name on the ballot as a candidate, it will be subject to rational basis, not strict scrutiny.

A state provided for the free distribution of textbooks to all public and private school students, certified teachers, and offered vouchers and tuition grants for families wishing to send their children to private schools. One private school that offers elementary and secondary education in the state denies admission to all children of Canadian descent. Two children whose family originated from Toronto, and who had lived in the state for several generations, were denied admission to the private school. The parents subsequently filed an action against the state challenging the distribution of vouchers and tuition grants to the school was unconstitutional. What of the following is the strongest argument in favor of finding the challenged public aid unconstitutional? (A) Issuing tuition grants to this private school furthers segregation. (B) It is unconstitutional for a state to provide aid to private schools. (C) The Constitution forbids private, as well as public, bias in education. (D) Tuition grants for private schools do not serve any legitimate educational function.

(A) Issuing tuition grants to this private school furthers segregation. SCOTUS has held that public aid given to private, segregated schools--including the free distribution of textbooks, exclusive use of public facilities, or the issuance of tuition grants--is unconstitutional [Norwood v. Harrison, 413 U.S. 455 (1973)]. The majority opinion stated that a state cannot give significant aid to schools that practice racial or other invidious discrimination.

On October 1, a seller mailed a letter to a buyer offering to sell a specified quantity of shirts at list price. The buyer received the seller's offer on October 2. The next day, the buyer mailed the seller a letter of rejection. The buyer then changed his mind and decided to accept the seller's offer. On October 4, the seller sent the buyer a letter revoking his original October 1 offer. On October 5, the buyer emailed the seller indicating that he wished to accept the seller's offer. The seller read the email on October 6. On October 7, the buyer received the seller's letter of revocation. The following day, the seller received the buyer's rejection. The seller subsequently refused to sell the shirts to the buyer, and the buyer sued for breach of contract. Assume that the buyer's email complies with the Statute of Frauds. Which of the following is most accurate? (A) Judgment for the buyer, because he accepted in a reasonable manner and before receiving notice of the seller's revocation. (B) Judgment for the buyer, because his acceptance was effective upon dispatch. (C) Judgment for the seller, because he revoked his offer before receiving the buyer's acceptance. You Answered (D) Judgment for the seller, because the buyer did not accept in a proper manner.

(A) Judgment for the buyer, because he accepted in a reasonable manner and before receiving notice of the seller's revocation. This is a classic case of the mailbox rule, but with a twist. Because the buyer sent a rejection prior to sending his acceptance, the mailbox rule will not apply, and so the acceptance will not be effective upon dispatch. Instead, the first communication to reach the seller will control. Therefore, as soon as the seller read the email from the buyer accepting the offer, the buyer's acceptance was effective.

One evening, the defendant was playing horseshoes with his son in the front yard of his house. One of the horseshoes went too far and landed on the neighbor's lawn, breaking the neighbor's bird bath. Immediately after hearing the crash, the neighbor stepped outside and began yelling about the bird bath. The neighbor said that the defendant's son was going to be a lousy sport, just like his dad. Infuriated, the defendant hurled the horseshoe in his hand at the neighbor's head. The horseshoe hit the neighbor in the temple, killing him instantly. Of what crime can the defendant be found guilty? (A) Murder, because the defendant's actions were malicious. (B) Murder, because the defendant was the initial aggressor. (C) Voluntary manslaughter, because the neighbor's insults caused the defendant to react violently. (D) Involuntary manslaughter, because the defendant did not intend to kill his neighbor.

(A) Murder, because the defendant's actions were malicious. All forms of murder require malice, such as intent to inflict serious bodily injury, wanton and willful misconduct, or depraved-heart murder. Here, there is no indication that the defendant intended to kill his neighbor. While his neighbor died as a result of the defendant's purposeful act, the act of throwing a horseshoe at someone could be construed as an attempt to kill, or even as an attempt to inflict serious bodily injury. As such, the defendant could be convicted of murder.

A man owned a home security business. In addition to selling personal defense devices, he also trained dogs to attack strangers at night. He sold and leased his guard dogs to various business and factory owners, who used the guard dogs to frighten away intruders from entering their premises at night. One evening, the man was in the backyard of his home training three of his guard dogs. The backyard was enclosed with a chain-link fence and a latched gate that prevented the dogs from running out. After the training session, the man opened the gate and permitted the dogs to run loose in his front yard. Minutes later, a bicyclist got a flat time in from of the man's home and was attacked by several of the dogs. The bicyclist suffered severe injuries and died as a result of the attack. At trial, it was conceded that the man's dogs attacked a postal worker two weeks prior to this incident. For what crime should the man should be found guilty? (A) Murder, because the man's actions were malicious. (B) Involuntary manslaughter, because the man did not command the dogs to attack the bicyclist. (C) Voluntary manslaughter, because the dogs could have reasonably believed the bicyclist was an intruder. (D) Reckless endangerment, because the dogs killing the victim was a superseding cause.

(A) Murder, because the man's actions were malicious. Depraved-heart murder occurs when a defendant's conduct creates an unjustifiable, very high degree of risk of death or serious bodily injury to another, even though unaccompanied by any intent to kill or to do serious bodily injury, and such conduct actually causes the death of another. Because the man had trained his guard dogs to attack at night and then opened the gate to let them run loose, he will be criminally responsible for the death of the bicyclist. This is so especially in light of the fact that he was aware that these dogs had previously attacked postman postal worker just two weeks prior to this incident.

A defendant had an argument with a patron in a bar over a game of pool. As they were quarreling, the defendant pulled out his knife, intending only to frighten the patron. The knife was six inches long, and the defendant held it out towards the patron. The patron immediately retreated from the pool table, telling the defendant that he was a hemophiliac and that he did not want any trouble. Wanting to have some fun at the patron's expense, the defendant drew near and swung the knife in the patron's direction. The defendant got a little too close to the patron and unintentionally nicked the patron's arm. The bar was far from a hospital, and by the time the patron arrived at the emergency room, he had died from his injury. What is the most serious crime for which the defendant could be convicted? (A) Murder. (B) Voluntary manslaughter. (C) Involuntary manslaughter. (D) Battery.

(A) Murder. While the defendant only intended to frighten the patron (which would be an assault) and did not intend to kill him or to inflict serious bodily harm, there is malice, because he was aware that the knife could cut the patron, who was a hemophiliac. As such, the defendant could be convicted of murder.

After an airplane crash at a small airfield, an inspector for the Federal Aviation Administration investigated the crash. In her initial visit, the investigator took pictures of the scene and made some notes after conducting interviews. One particular interview was with a novice pilot who happened to be at the airfield when the crash occurred. He recalled to the investigator seeing something wrong with the left wing of the crashed plane before it took off, but that he didn't have a chance to warn the pilot. He went on to say that "even a novice like him would have done a quick safety check and spotted the issue before taking off." The investigator then compiled her findings and submitted them in a report. Her conclusion was that the pilot failed to conduct himself as a reasonably prudent pilot. A ground worker at the airfield who had been burned trying to rescue the pilot brought a civil case against the pilot's estate. The ground worker wanted to introduce the novice pilot's statement from the investigator's report. The estate's attorney objected. Is the novice pilot's statement admissible? (A) No, because it is hearsay not falling within any exception. (B) No, because the estate does not have an opportunity to cross-examine the investigator or the novice pilot. (C) Yes, as a business record, because the FAA routinely conducts such investigations. (D) Yes, because it is contained inside of a public record compiled by a government agent under duty of law.

(A) No, because it is hearsay not falling within any exception. Rule 803(8) creates a hearsay exception for most public records and reports. Properly certified official records from public offices are generally admissible if they are routine, factual, based on personal knowledge of public officials, and appear reasonably reliable. Investigative reports, reports with recommendations, and one-time reports prepared for a narrow purpose are generally admissible except against a defendant in a criminal case. No witness is required if the document is certified; it will be considered self-authenticated under Rule 902. However, statements by private citizens may not be included in official records unless they separately fall within a hearsay exception [D.W.S. v. L.D.S., 654 N.E.2d 1170 (Ind. Ct. App. 1995) (report from welfare department based on interviews of people who were under no duty to report did not qualify; person making report must be public employee with personal knowledge)]. Here, because the statement of the novice pilot is not independently admissible, it will not be admitted simply because it was written in a public record.

A famous football player had his dog stolen from his yard. He immediately went to social media and implored anyone with details to contact him. He put a description of the dog, where it was last seen, and said that he would be forever indebted to whomever found his dog. One particular fan immediately went out looking for the dog in the area surrounding the player's home. A few days passed and the player became desperate, so he offered a $5,000 reward for finding the dog. After several days of looking, the fan finally saw the dog tied to a bench. He untied the dog and went onto the player's website to contact him. The fan then saw the reward offer and returned the dog, demanding the reward. The player refused to pay the fan the $5,000. Is the player liable to pay the fan the reward money? (A) No, because the fan did not know of the reward before he found the dog. (B) No, because the player was unaware that the fan had accepted the offer. (C) Yes, because the fan found the dog as requested. (D) Yes, because the fan learned of the reward before he returned the dog.

(A) No, because the fan did not know of the reward before he found the dog. A unilateral contract is created when an offeror seeks acceptance by performance, rather than by a promise in return. An offer in a unilateral contract cannot be revoked once performance has begun. Here, however, the fan undertook performance not based on the reward money, but based on the request for help. Further, the fan did not learn of the reward offer until after he had already performed by finding the dog. Therefore, his performance did not act to accept the player's unilateral offer.

A border patrol officer found packages of heroin glued between the soles of gym shoes being exported from Colombia into the United States. Narcotics agents arranged for a controlled delivery of the boxes to the address listed on the packages. After the boxes were delivered to the address, the house was put under surveillance. One week later a woman driving a car with expired plates arrived at the address and began loading the boxes into the trunk of the car. As she drove away the car was stopped by the narcotics agents and the driver was arrested. The agents searched the entire vehicle without first obtaining a warrant. The box was opened and the heroin was found concealed in the soles of the gym shoes. At her trial on charges of possession of heroin and conspiracy, the defendant moved to suppress the search of the trunk. Should the motion be granted? (A) No, because the officers had probable cause to search the trunk. (B) No, because the search was conducted incident to a lawful arrest. (C) Yes, because the warrantless search was limited to the wingspan of the driver. (D) Yes, because the agents entrapped the defendant by arranging the controlled delivery.

(A) No, because the officers had probable cause to search the trunk.

Police officers received a tip from a reliable informant that a night club was operating a drug distribution business out of the basement in the building where the club was located. During a police surveillance of the building, the officers observed a patron leaving the club by the basement stairs and placing what appeared to be a weapon inside his raincoat. The officers stopped the patron on the street who denied that he was ever in the basement of the night club. At that point the officer, fearing for his safety, patted down the outer clothing and felt small tube-shaped vials in the defendant's shirt pocket. The defendant was placed under arrest. The substance in the vials field-tested positive for cocaine. The defendant filed a motion to suppress the cocaine. At the suppression hearing, the arresting officer testified that based on his 20 years of experience, he concluded that the tube-shaped objects were glass vials used by drug dealers to sell narcotics. If the court believes the officer's testimony, should the evidence of the cocaine be suppressed? (A) No, because the stop was reasonable in duration and the officer had a reasonable suspicion that the defendant was armed and dangerous. (B) No, because the officer had probable cause to believe that crime had been committed. (C) Yes, because the scope of a pat down in a Terry stop is limited to a search for weapons. (D) Yes, because the informant supplied no information about the patron to police before the stop.

(A) No, because the stop was reasonable in duration and the officer had a reasonable suspicion that the defendant was armed and dangerous. SCOTUS held in Terry v. Ohio [392 U.S. 1 (1968)] that a police officer may legally stop a person without probable cause if the officer has a reasonable and articulable suspicion of criminal activity. If at the time of the stop, the officer reasonably believes that the person may be armed and dangerous, the officer is authorized to conduct a protective frisk to locate any weapon that may endanger the officer's safety. The scope of a protective frisk is limited to a pat-down of the defendant's outer clothing. However, the Supreme Court also held in Minnesota v. Dickerson [508 U.S. 366 (1993)] that the officer may, during the course of a protective frisk, reach inside the defendant's clothing and seize any item that the officer reasonably believes is a weapon or contraband based on its "plain feel." This is known as the plain feel doctrine.

A fitness instructor wanted to open a new gym in a city neighborhood known locally as "Fitnessville," because of its many gyms, fitness studios, and juice bars. The man purchased an empty lot and submitted the appropriate plans to the city authorities for a building permit. The building inspector, backed by the city zoning and planning commission, denied the instructor's application for a building permit, stating the following reasons in a written decision: (1) the local neighborhood could not sustain any further traffic entering and exiting the area; (2) the neighborhood already had 12 gyms of a similar nature, four of which would be on the same street and block as the proposed gym location; and (3) the location of the entrance to the planned gym was too close to a dangerous intersection. The fitness instructor requested an opportunity before the city zoning and planning commission to address each of these reasons for denying his building permit, but the city authorities replied that the decision was final. The fitness instructor then brought an appropriate action challenging the constitutionality of the city's decision. Which of the following provides the fitness instructor with the strongest argument? (A) Procedural due process. (B) The Contract Clause. (C) The Privileges and Immunities Clause of the 14th Amendment. (D) The Takings Clause

(A) Procedural due process. Under the 5th & 14th Amendments, citizens are protected against the deprivation of life, liberty, or property without the due process of the law. Procedural due process guarantees notice & the right to be heard.

A man has been arrested and charged with homicide for the intentional killing of a neighbor. At trial, the prosecution argued that the man committed murder, and the defendant, in turn, claimed that he should be found guilty of voluntary manslaughter. Regarding voluntary manslaughter, which of the following statements is most accurate? (A) Provocation is adequate if it would cause a reasonable person to lose self-control. (B) Provocation is adequate if it would cause the defendant to lose self-control. (C) As long as the man was still laboring under the heat of passion from the incident, it is irrelevant whether a reasonable person would have cooled off. (D) A showing by the man that he was intoxicated would reduce a charge of murder to manslaughter.

(A) Provocation is adequate if it would cause a reasonable person to lose self-control. Voluntary manslaughter involves the intentional killing of another in the heat of passion brought about by adequate provocation. Provocation is adequate if it would cause a reasonable person to lose self-control.

A seller sent an email to a potential buyer offering to sell his patio furniture to her for $5,000. The buyer immediately responded via email asking whether the offer included the umbrella that was sitting in the garage. The seller emailed back: "No, it does not; just what is sitting on the patio. If you want an umbrella, you will have to buy one." The buyer then ordered an umbrella that would fit on the table and matched the color of the chairs. Later that day, the buyer replied to the seller: "I accept your offer." The seller then wrote back, "I have changed my mind, I've decided to keep the furniture." If the buyer sues the seller claiming there was a valid contract, who is likely to prevail? (A) The buyer, because she accepted the seller's offer within a reasonable amount of time and before the seller revoked his offer. (B) The buyer, because her ordering the umbrella is a sufficient act in reliance on the seller's offer. (C) The seller, because the offer lapsed after the buyer did not accept the original offer. (D) The seller, because the buyer's initial email was a counteroffer which terminated her power of acceptance.

(A) The buyer, because she accepted the seller's offer within a reasonable amount of time and before the seller revoked his offer. The UCC governs contracts for the sale of goods. An offer was made by the seller via email. It was not a firm offer, because it did not specify that it would remain open for a period of time. Therefore, it is revocable either through lapse of time or a rejection or counteroffer. It appears from the facts that this conversation is taking place on the same day, so the offer would not be revoked due to lapse of time. The buyer notified the seller of her intent to accept before the seller revoked the offer, and within a reasonable time. Therefore, a contract was formed.

The owner of a lakefront house had leased the house to a renter for the past three summers. Last year, on October 1, the owner sent the renter a letter stating, "Because you have been such a great tenant, I will rent you the house on the same terms as last year. However, I need your answer by February 1." The renter, uncertain as to when he would be taking his annual vacation, decided to put the letter aside for the time being, intending to respond to it by February 1. On January 15, the owner received an offer from the renter's cousin to rent the house at a higher price than the renter had paid the prior year. The owner accepted. On January 20, after learning about his cousin's rental, the renter telephoned the owner and said, "I have decided that I want the house for the summer." The owner replied that he had already accepted an offer to rent the house from the renter's cousin. The renter then filed a lawsuit seeking to enforce the contract. If the renter is unsuccessful in his lawsuit, which of the following provides the best reason? (A) The contract with the cousin indirectly revoked the offer to the renter. (B) The contract with the cousin terminated the offer to the renter. (C) There was no mutual assent, because the renter was unsure whether he wanted to vacation at the lake. (D) A common law offer cannot extend beyond three months without consideration.

(A) The contract with the cousin indirectly revoked the offer to the renter. An offer is a manifestation of intent by the offeror to be bound by the contract that is communicated to the offeree with definite and certain terms. Here, the owner wrote, "I will rent you the house on the same terms as last year. I need your answer by February 1," thereby demonstrating his intent to be bound by a contract. Although the letter itself did not specify the terms, the reference to "the same terms as the previous year" is sufficiently definite to form an enforceable offer. The owner's letter also included the stipulation that he must have a response from the renter by February 1. Generally, promises to keep an offer open are enforceable and irrevocable as long as the offeree pays separate consideration for the option. In this case, the renter did not pay separate consideration for the option, and, as such, the owner was free to revoke the offer at any time before February 1. When the renter learned of the fact that the owner had rented the lakefront house to his cousin, this indirectly revoked the offer to the renter, and he could no longer accept the offer on January 20.

A police officer saw a van that had a bumper sticker with the symbol of a rock band that is known to have fans that regularly take recreational drugs during the concert as a way to enhance their enjoyment of the show. The van entered a restaurant parking lot and the driver and her friend walked into the restaurant. The policeman decided to search the van for drugs. He opened the van door in the back which was unlocked and started looking inside. He saw a bag of marijuana. The police officer arrested both the driver and her friend when they came back. What is the most likely outcome of the driver's and her friend's motions to suppress the evidence? (A) The driver's motion will be granted, but the friend's will not. (B) The friend's motion will be granted, but the driver's will not. (C) Both motions will be granted as to the marijuana, even though the marijuana was in plain view. (D) Neither motion will be granted.

(A) The driver's motion will be granted, but the friend's will not. The search was clearly illegal in that the officer did not obtain a warrant and there were no exigent circumstances or other applicable exceptions to the warrant requirement. Thus, the driver's motion to suppress the marijuana obtained during the search of her van will be granted. The driver's friend, however, has no standing to challenge the search of the driver's van, and, therefore, the marijuana will not be excluded as against her.

A defendant is wandering down the street in a drunken stupor, periodically stepping off the sidewalk onto the street and into the path of oncoming traffic. A police officer saw the defendant and decided he should help. The officer stopped the defendant and asked his name and how he was doing. The defendant mumbled something about how he "loved her" and how he "shouldn't have done it." Figuring that the defendant was simply drunk, the officer ignored his remarks and decided the best course of action would be to take away any remaining alcohol from the defendant so that he could sober up. The officer reached into the defendant's coat pockets and, to his surprise, found a small pistol. "What are you doing with this, man?" the officer asked. When the defendant did not reply, the officer became angry. "I asked you a question," the officer said. "You'd better answer it." The defendant then told him that he had killed his girlfriend and dumped her body in a drainage ditch. The officer arrested the defendant and read him his rights. At the defendant's murder trial, the state moved to admit the evidence of the gun. How is the court likely to rule? (A) The evidence will be suppressed because the officer had no reason to believe that the defendant was armed. (B) The evidence will be suppressed because the Terry stop of the defendant was illegal. (C) The evidence will be admitted because there was reasonable suspicion for the Terry stop. (D) The evidence will be admitted because guns constitute a threat to public safety.

(A) The evidence will be suppressed, because the officer had no reason to believe that the defendant was armed. A Terry stop has 2 parts: the stop & the frisk. For a constitutional stop, an officer must have a reasonable suspicion that criminal activity may be "afoot." During the course of that stop, if the officer has a reasonable and articulable suspicion that the suspect is armed and dangerous, he may conduct a pat-down frisk search for concealed weapons. Here, the officer's Terry stop of the defendant was likely valid, in that the defendant's drunken weaving along the sidewalk was bizarre enough to suggest that criminal activity was afoot. However, following this stop, there was nothing to suggest that the defendant was armed and dangerous. By reaching into the defendant's pocket, the officer conducted an actual, and unlawful, search.

A man is walking down the street and is weaving down the street in a drunken stupor. A police officer saw the man and asked, "Do you need help?" The man, who reeked of alcohol, said, "Tired. I am going home." The officer ignored his remarks and decided the best course of action would be to take away any remaining alcohol from the defendant so that he could sober up. The officer reached into the defendant's coat pockets and, to his surprise, found a small pistol. "What are you doing with this, man?" the officer asked. When the defendant did not reply, the officer became angry. "I asked you a question," the officer said. "You'd better answer it." The defendant then told him that he had robbed a liquor store. The officer arrested the defendant and read him his rights. At the defendant's robbery trial, the state moved to admit the evidence of the gun. How is the court likely to rule? (A) The evidence will be suppressed because the officer had no reason to believe that the defendant was armed. (B) The evidence will be suppressed because the Terry stop of the defendant was illegal. (C) The evidence will be admitted because there was reasonable suspicion for the Terry stop. (D) The evidence will be admitted because guns constitute a threat to public safety.

(A) The evidence will be suppressed, because the officer had no reason to believe that the defendant was armed. A Terry stop has two parts: the stop and the frisk. For a constitutional stop, an officer must have a reasonable suspicion that criminal activity may be "afoot." During the course of that stop, if the officer has a reasonable and articulable suspicion that the suspect is armed and dangerous, he may conduct a pat-down frisk search for concealed weapons. Here, the officer's Terry stop of the defendant was likely valid, in that the defendant's drunken weaving along the sidewalk was bizarre enough to suggest that criminal activity was afoot. However, following this stop, there was nothing to suggest that the defendant was armed and dangerous. By reaching into the defendant's pocket, the officer conducted an actual, and unlawful, search.

A state-owned vast tracts of woodlands containing highly desirable lumber, and provided licenses to businesses to exploit these resources, with various regulations in place to ensure that a certain percentage of them would be preserved and the woodlands would not be completely depleted. State building codes required that a minimum of 10% of the lumber used by construction companies be the highly desirable, locally sourced lumber, and in-state commercial enterprises could obtain a license from the state to cut timber up to a set amount for an annual fee of $7,500. Out-of-state commercial enterprises, which made up only about a small fraction of the lumber industry in the state, could obtain a license for an annual fee of $20,000 but were permitted to remove 10% more lumber than in-state businesses, a difference which enabled those nonresident enterprises to make a profit greater than the difference in license costs. The schemes were adopted after research showed that they would keep the level of exploitation at or below the rate of regrowth without requiring the state to resort to a lottery system for available licenses, which would have been administratively costly and burdensome. A nonresident commercial enterprise sued the state claiming that the disparities in the licensing and regulatory scheme were unconstitutional. How should the court rule on the state's resource management system? (A) The license system is constitutional because it is within the police power of a state to regulate and preserve state-owned natural resources. (B) The license system is constitutional because there is a compelling state interest. (C) The license system is unconstitutional because it violates the Equal Protection Clause of the Fourteenth Amendment. (D) The license system is unconstitutional because it violates the Privileges and Immunities Clause of Article IV, Section 2.

(A) The license system is constitutional, because it is within the police power of a state to regulate and preserve state-owned natural resources. In Baldwin v. Montana Fish and Game Commission [436 U.S. 371 (1977)], the U.S. Supreme Court held that the protection of the wildlife of a state is peculiarly within the police power of the state. Furthermore, the Court later held that discrimination against nonresidents will be given special consideration if its purpose is the preservation of natural, state-owned resources [Sporhase v. Nebraska, 458 U.S. 941 (1982)]. Because the purpose of the disparate licensing schemes was to allow the limited exploitation of natural resources, these schemes will likely be found constitutional.

An environmental activist group vehemently opposed the planned expansion of an oil pipeline that would take it through a wildlife preserve. Despite long periods of protests and media coverage, the oil company was nevertheless granted all of the necessary state and federal permits and permissions to go ahead. Deciding that nonviolent protests were not enough, a splinter faction of the environmentalist group decided to take a more hands-on approach. Late one night when nobody was present, the faction detonated blasting caps on a nearby mountain that caused a small avalanche above the oil company's supply cache. The falling rocks destroyed material and machinery and blocked necessary roads into and out of the wildlife preserve, costing the oil company millions of dollars and months of delay while it had to clear and rebuild many roads. Thereafter, the state legislature enacted a law restricting the rights of members of the environmentalist group in several ways, one of which was excluding members of the group from holding elected office in the state or working for a government agency or contractor. A representative justified the legislation on the grounds that "ours is a peaceful state, and these verminous scum clearly have no place here." Several members of the group who did not participate in the blasting were soon informed that they would no longer be allowed to remain in elected positions, including a city council member and a county school board official. If the members challenge the constitutionality of the legislation in court, what is the likely outcome? (A) The members will prevail because the law is a bill of attainder. (B) The members will prevail because they did not engage in violence as a member of the group. (C) The state will prevail because it acted within its police power. (D) The state will prevail because the group is a subversive organization.

(A) The members will prevail, because the law is a bill of attainder. A bill of attainder punishes named individuals or easily ascertainable members of a group without the benefit of judicial trial. Article I, Section 9, Clause 3 of the U.S. Constitution states: "No bill of attainder...shall be passed"; and Article I, Section 10, Clause 1 provides: "No state shall pass any Bill of Attainder." These apply to states as well as the federal government. The state legislation in this question is a bill of attainder because it amounts to legislative punishment for membership in the environmental activist group. Members of the group, such as those in question, are prohibited from holding elective office or being employed by the government or government contractors without a judicial determination of their right to serve. The legislation is therefore unconstitutional.

A man decided to take his own life and went to the roof to jump to the street below. On his way up the stairs, he told his neighbor what he was planning to do. The neighbor had a strong belief that anyone who committed suicide would not enter Heaven. Therefore, he snuck up behind the man as he was readying to jump off the rooftop. After the man jumped, the neighbor shot him once in the back before the man struck the sidewalk below. At trial, the medical examiner testified that the man died of a gunshot wound, and that in all likelihood, the fall would not have killed the man. The jury also found that the neighbor's religious belief was honestly held. If the neighbor is prosecuted for murder, what should the result be at trial? (A) The neighbor should be convicted, regardless of the man's intent or the medical examiner's testimony. (B) The neighbor should be convicted, because the medical examiner testified that the fall would not have killed the man. (C) The neighbor should be acquitted, because the man intended to kill himself. (D) The neighbor should be found guilty of a lesser charge, because the jury believed that his religious belief was honestly held.

(A) The neighbor should be convicted, regardless of the man's intent or the medical examiner's testimony. Murder requires malice. Malice can be express or implied. Here, there is express malice, because the defendant intentionally took the life of another. Regardless of whether the man would have died or not otherwise is irrelevant, because it was the neighbor's act that caused the man's death. Moreover, the fact that the neighbor was acting in what he thought was the best interest of the victim is also irrelevant.

A police officer has long been suspicious of one of his neighbors. Every time he encounters her, she behaves furtively and seems nervous. Convinced that there is a reason for the neighbor's edginess, the officer decides to investigate further. He knows that the neighbor lives alone, so when she leaves the house with friends one evening, he walks into the open garage and tries the door to the house. Finding it unlocked, he enters the house. In plain view on the coffee table is a large quantity of cocaine and a clear plastic bag containing cash. Looking further, the officer finds records tying the neighbor's sister to the drug operation. What is the most likely outcome of the neighbor's and her sister's motions to suppress the evidence? (A) The neighbor's motion will be granted, but the sister's will not. (B) The sister's motion will be granted, but the neighbor's will not. (C) Both motions will be granted as to the records, but not as to the items that were in plain view. (D) Neither motion will be granted.

(A) The neighbor's motion will be granted, but the sister's will not. The search was clearly illegal in that the officer did not obtain a warrant and there were no exigent circumstances or other applicable exceptions to the warrant requirement. Thus, the neighbor's motion to suppress all of the evidence obtained during the search of her house will be granted. The neighbor's sister, however, has no standing to challenge the search of the neighbor's house, and, therefore, the items will not be excluded as against he

A resident alien was recently certified to be a teacher by a state regulatory agency. She applied to several schools that were hiring in her district, but all of them refused to hire her, citing a state law prohibiting non-citizens from teaching in a public school. The woman contacted the schools and was told that, if not for the state law, they would have hired her. The woman sued the state to challenge the constitutionality of the law prohibiting non-citizens from public school teaching. What is the most likely outcome? (A) The statute is constitutional, because public school education involves the functioning of government. (B) The statute is constitutional, because the exclusion is rationally related to legitimate state purposes. (C) The statute is unconstitutional, because the action of the state violates the Privileges and Immunities Clause of Article IV of the Constitution. (D) The statute is unconstitutional, because there is not sufficient justification to discriminate against aliens as a class.

(A) The statute is constitutional, because public school education involves the functioning of government. The state may discriminate against aliens only in specific matters relating to the governmental process, such as the denial of a right to run for elective office/to hold a position as a police officer/probation officer/a public school teacher. In all other cases, a state's attempt to discriminate against persons based on alienage will be subject to the strict scrutiny test, because alienage is a suspect classification.

The United States Post Office received an anonymous tip that heroin would be found inside a large black hat box sent to a local address from Panama. Pursuant to this tip, United States postal employees opened the hat box bound for the local address and found heroin inside. They then resealed the package and left the heroin inside. The local police were notified and watched as the package was delivered to the address. The police then secured a warrant to search the house for the package. About two hours after the package was delivered, the warrant was executed. The man who opened the door was arrested, and the officers found the package unopened, sitting on the kitchen counter. After seizing the package, the officers looked through the rest of the house. In an upstairs cedar chest, they found a similar hat box that also contained heroin. At trial, the man brings a motion to suppress the heroin found in the cedar chest. Should the court grant the motion? (A) Yes, because the search exceeded the authority granted by the warrant. (B) Yes, because the initial search by the United States Post Office was without probable cause. (C) No, because, having found the package, the officers had probable cause to believe more narcotics could be located in the house. (D) No, because the heroin was found pursuant to a valid protective sweep.

(A) Yes, because the search exceeded the authority granted by the warrant. The warrant was valid, but its validity was triggered by and limited to the delivered package [United States v. Grubbs, 547 U.S. 90 (2006)]. Accordingly, once the only object of that search was discovered, the warrant did not authorize a further exploratory search of the house [Horton v. California, 496 U.S. 128 (1990)].

After driving his car up a curb and hitting a streetlight, an accident for which he was found to have been at fault after a trial, a 70-year-old retiree had his driver's license automatically revoked pursuant to a state law requiring all drivers over the age of 65 who are involved in a serious accident to have their licenses revoked until they complete a safety course and pass another driving test. The retiree was a handyman who now volunteered by going to the homes of people who could not afford to hire someone to make various repairs. The man was now unable to help these indigent people due to the revocation of his license. He brought suit against the state challenging the automatic revocation of his license without his first having an opportunity to present evidence on the accident to the state's department of motor vehicles. How should the court rule? (A) Against the retiree, because he is now retired. (B) Against the retiree, because he received adequate due process. (C) For the retiree, because a pre-termination hearing was required before his license could be revoked. (D) For the retiree, because his Fourteenth Amendment rights were violated.

(B) Against the retiree, because he received adequate due process. The deprivation of a driver's license is a property interest protected by the Fourteenth Amendment [Bell v. Burson, 402 U.S. 535 (1971)]. However, the Supreme Court has found that there is a substantial public interest in administrative efficiency which would be impeded by the availability of a pre-termination hearing in every case. Especially when there is an opportunity for the validity of the plaintiff's fault to be determined in a prior setting, the automatic suspension or revocation of a driver's license does not violate a plaintiff's procedural due process rights when weighed against the important public interest in safety on the roads and highways, and the prompt removal of a safety hazard [Dixon v. Love, 431 U.S. 105 (1977)]. As such, the man's procedural due process rights were not violated in this case.

On December 1, Arnold, who had purchased two tickets for a holiday magic show, sent the following email to his three friends, Baker, Charlie, and Dora: "Dear Baker, Charlie, and Dora: I have two tickets for the upcoming holiday magic show. If any of you are interested in buying my tickets, I will let you have them for a reasonable price, but I must have your reply by December 20. s/Arnold" The tickets, which cost Arnold $10, were now selling for between $50 and $70. On December 19, Arnold received an email from Baker that stated, "I accept your offer and will pay you $50 for the two tickets." Arnold did not immediately respond to Baker's email. On December 22, Arnold received the following text message from Charlie: "I am dying to go to the concert...will pay $70 for your seats." The next day, Arnold sent a reply text to Charlie which read: "The tickets are yours. You may take delivery upon payment of the $70." Dora did not respond to Arnold's letter. Baker tendered the $50 to Arnold within a reasonable time, but Arnold refused to sell the tickets to him. In an action by Baker against Arnold for breach of contract, judgment should be for whom? (A) Arnold, because his email of December 1 and Baker's reply were too indefinite to constitute an offer and acceptance. (B) Arnold, because his email was not an offer, but rather, an invitation to bargain, and he was free to accept either Baker or Charlie's offer. (C) Baker, because he accepted Arnold's offer first. (D) Baker, because Arnold's offer lapsed on December 20, and therefore, Charlie's attempted acceptance was invalid.

(B) Arnold, because his email was not an offer, but rather, an invitation to bargain, and he was free to accept either Baker or Charlie's offer. An offer is the manifestation of a willingness to enter into a bargain, so made as to justify another person in understanding that his assent to that bargain is invited and will conclude it. Arnold's email to three people does not qualify as an offer. An offer must evidence a commitment by the offeror that he reasonably intends to be bound upon acceptance by the offeree, rather than a mere statement of present intention or preliminary negotiations. Arnold's email stated, "I will let you have [the tickets] for a reasonable price." This language evidences an intent to invite a reasonable offer by the message's recipients. Therefore, Arnold's email was not an offer. On the other hand, Baker's email was an offer, which Arnold chose not to accept. Charlie's text was also a valid offer and, in this case, Arnold accepted it upon dispatch of his email on December 23. Therefore, Baker will not prevail against Arnold.

The police received a report that women dressed as superheroes had robbed a man in his house and stolen many of his valuables. The only way onto or off the island where he lived was a ferry. The last ferry was getting ready to leave for the night when the report came in to the police. Without attempting to get a warrant, the police stopped all the cars waiting for the ferry. In one vehicle, the police noticed three women fidgeting rather nervously. Upon searching their auto, the police found the victim's stolen property hidden under the front seat. The defendants were immediately placed under arrest. At their trial for armed robbery, the defendants move to suppress the use of the property confiscated by the police as evidence. What should the ruling be? (A) Granted, because the police did not have a warrant to search their car. (B) Granted, because the police did not have probable cause to suspect that their car contained evidence of the crime. (C) Denied, because the police had probable cause to search their car and, under the circumstances, no warrant was required. (D) Denied, because even though the detention was unlawful, the police had reasonable suspicion to believe that their car contained evidence of the crime since they observed the women fidgeting nervously.

(B) Granted, because the police did not have probable cause to suspect that their car contained evidence of the crime. Keep in mind that the warrant requirement is central to the 4th Amendment protection against unreasonable searches and seizures. As a basic rule, all warrantless searches are unconstitutional unless they fall into one of the following seven exceptions to the warrant requirement: (1) search incident to a lawful arrest; (2) the "automobile" exception; (3) plain view; (4) "stop and frisk"; (5) consent; (6) hot pursuit; and (7) other emergencies. A valid warrantless search must meet the requirements of at least one of the above exceptions. With respect to the "automobile" exception, the police must have probable cause to suspect or reasonably believe that the vehicle contains evidence of the crime. In the present example, the police did not have a reasonable or articulable suspicion to believe that the defendants' car contained evidence of the crime. As a result, choice (B) is correct because the police did not have probable cause to conduct a warrantless search of the auto. Choice (A) is wrong because, if the police had probable cause to suspect that the vehicle contained evidence of the crime, then no warrant would have been necessary. Choice (D) is incorrect because the mere fact that the police observed the women "fidgeting nervously" would not by itself give the officers probable cause to suspect that the defendants' car contained evidence of the crime. Choice (C) is incorrect because the police did not have probable cause to search the vehicle, for reasons stated above.

A man lived in a duplex and worked at night as a security guard. The man soon became annoyed at his upstairs neighbor, who kept a large number of dogs inside his home during the day while he was at work. The noise from the dogs was so loud that the man could not fall asleep. On several occasions, the man asked the neighbor to do something about the dogs, but the neighbor simply replied, "Get a day job!" The following day, the man was coming home just as the neighbor was leaving for work and again pleaded with him to keep the dogs quiet, because he had been awake for 24 hours straight at that point, and desperately needed sleep. The neighbor simply smiled and got into his car, but said nothing to the man. A short time later, the dogs began to bark, and the man fired three shots through the ceiling of his apartment, as at that point he merely wanted to put an end to the noise. One of the bullets struck the neighbor, who had returned home because he had forgotten his laptop. The neighbor died a week later from the gunshot wound. For what crime should the man be found guilty? (A) Murder, because the man intentionally shot into a dwelling. (B) Involuntary manslaughter, because while the defendant was criminally negligent, he did not act with malice. (C) Voluntary manslaughter, because a jury could conclude the man was adequately provoked. (D) Felony murder, because the man intended for the bullet to enter the apartment to cause either damage to the building or injury to the dogs.

(B) Involuntary manslaughter, because while the defendant was criminally negligent, he did not act with malice. The man lacked malice, because he was unaware that anyone was upstairs at the time. While it is not reasonable to shoot a gun into a dwelling, occupied or not, the defendant was under the impression that the victim had left the premise. Therefore, while his action was criminally negligent and led to the death of the neighbor, the man lacked malice, and should be convicted of involuntary manslaughter.

A private investigator who does a great deal of divorce and pre-divorce work recently obtained evidence that a wealthy and volatile husband had been cheating on his wife. When the husband learned of the investigator's discovery of the evidence, he came to the investigator's office and initiated a series of screaming matches with him. The investigator always believed the husband to be an idiot because, even though he had several inconspicuous cars at his disposal, he always drove an absolutely unique classic car when he went to meet his lover. One night, when the investigator was crossing the street, he saw the husband's car coming directly at him. When the car came within 20 feet of him without slowing down or swerving, the investigator pulled out his gun and shot at the husband. The husband died from the gunshot wound the next day. Prior to his death, the husband was heard to repeatedly say, "Why did that man shoot me? He could have gotten out of the way. My brakes failed, and I couldn't stop." The investigator, in turn, told the police that he thought that the husband was trying to hit him with the car to get revenge. Of what crime, if any, is the investigator likely to be convicted with regard to the husband's death? (A) Murder, because the investigator intended to cause the husband's death. (B) Manslaughter, because the investigator thought that the husband was trying to kill him. C) Murder, because the investigator acted with wanton and willful disregard for human life. (D) No crime.

(B) Manslaughter, because the investigator thought that the husband was trying to kill him. While self-defense is a full defense to murder, the force used in self-defense must be appropriate to the threat. Here, the facts indicate that the investigator did genuinely believe that his life was in danger from the husband. However, the facts also indicate that this belief was unreasonable and that the force used by the investigator to protect himself--shooting the husband with a gun--was unreasonable, given that merely moving out of the car's path would have sufficed to save him from harm. As such, the investigator's unreasonable belief that his life was being threatened by the husband constitutes an imperfect claim of self-defense, which would not operate to completely negate the investigator's liability but would serve to mitigate his murder conviction to manslaughter. As such, this is the best response.

A man was recently hired as a professor at a state university. The university had a policy of offering tenure to its professors after three years. Prior to that, the professor could be let go for any reason, though the university typically offered notice and conducted a hearing prior to terminating any professor's employment. 2.5 years after he was hired, the university administration underwent a change of personnel, and during this period, the professor was told that he was being let go. The professor was never given notice or a hearing prior to his termination. If the professor sued challenging the constitutionality of the state's action, will the professor succeed? (A) No, because a state's procedure with respect to state employees is a matter reserved to the state under the 10th Amendment. (B) No, because he does not have a right to notice and a hearing protected by the Due Process Clause of the 14th Amendment. (C) Yes, because he was denied a notice & hearing. (D) Yes, because the state's action is an act of attainder.

(B) No, because he does not have a right to notice and a hearing protected by the Due Process Clause of the 14th Amendment. An employee who has no expectation of continued employment (i.e., one who has been hired on an at-will basis) has no due process rights regarding termination. Here, because professors have no expectation of a permanent job until they are offered tenure after three years, they have no property right, and therefore, no entitlement to a hearing. As such, the state's discharge procedure need not be consistent.

In the wake of a recent presidential election mishap that garnered national media coverage, a state revamped its voting procedures in an attempt to streamline the process. Among the many changes the state instituted were vote-by-mail, same-day voter registration, and the requirement that voters check in (or register) at the table labeled with their party affiliation. One couple in the state, who were very active within their local community, were registered Republicans in a heavily Democratic district. The couple did not wish for their party affiliation to be made public, fearing repercussions. The couple challenged the constitutionality of the rule requiring voters to check in or register according to party affiliation. Assuming proper standing, should the court invalidate the rule? (A) No, because it passes strict scrutiny. (B) No, because it passes the rational basis test. (C) Yes, because it violates their procedural due process rights. (D) Yes, because it violates their right to privacy.

(B) No, because it passes the rational basis test. The court is likely to uphold this rule, because the state will only need to show a rational relationship between the rule and the objective to streamline voting procedures. Strict scrutiny applies when the government restricts the right to vote, a right that has been held to be fundamental under the Constitution. However, rational basis scrutiny applies to reasonable restrictions based on things like age or duration of residency. Moreover, voter registration requirements and regulation of the time, place, and manner of casting ballots are valid so long as they do not impose an undue burden on the right to vote. Here, there is no restriction at all on the right to vote, other than the concern that others will know your party affiliation. Because the state made vote-by-mail an option for everyone and because voter registrations are public records anyway, it seems very unlikely that this rule will be held to violate the Constitution. There is no constitutionally protected privacy right as to party affiliation.

A state enacted a statute which required that 60% of the employees of any business that was located in the state and that employed more than 10 people be residents of the state. A woman worked at a telemarketing firm close to the border in the state and commuted from the neighboring state. The telemarketing firm had 20 employees, 15 of whom lived outside of the state and either commuted into the office or worked remotely. The woman was informed by her manager that she would be one of the seven nonresident employees whose employment was being terminated pursuant to the new state statute. The woman properly filed an action challenging the state statute on constitutional grounds. Is the state statute constitutional? (A) No, because it authorizes employers to arbitrarily dismiss some nonresidents while retaining others. (B) No, because it unfairly discriminates in favor of residents of the state as to the basic right to employment. (C) Yes, because a state is permitted to treat nonresidents differently from residents whenever it sees fit. (D) Yes, because the woman was discharged in compliance with the ordinance and was not otherwise discriminated against by her employer.

(B) No, because it unfairly discriminates in favor of residents of the state as to the basic right to employment. The state is discriminating against nonresidents with respect to one of the privileges and immunities of citizenship protected by Article IV, Section 2--the right to pursue a livelihood. This clause protects nonresidents from being discriminated against by state or local governments with respect to certain basic rights or essential activities of citizenship. Among these are the right to own property, equal access to medical care and the courts, and the right to earn a living. Here, the state statute discriminates against nonresidents. When discrimination against the employment rights of nonresidents is found to exist, the law will be struck down unless the responsible state or local government can show such discrimination is proper because the nonresidents are a peculiar source of the evil the law was designed to redress. For instance, if the state were experiencing high unemployment because nonresidents were taking all the jobs, a court could find that the ordinance was constitutional. However, no such showing has been made here.

In the wake of the federal government suspending welfare benefits and subsidized medical care, states began to see a significant influx in citizens claiming such benefits under state programs. One state in particular, which provided better coverage than most others, soon found that many people were moving into the state for the sole purpose of claiming these benefits. In response to the suddenly increased drain on the state budget, the legislature passed a bill requiring that anyone filing a claim for welfare or state-subsidized medical care show proof of having resided within the state for at least one year prior in order to be eligible for such benefits. A woman moved to the state six months ago for a job, but was currently unemployed after being fired due to downsizing. The woman applied for state welfare benefits, but her application was denied because she had not been in the state for a full year. The woman filed a proper suit challenging the constitutionality of the statute. Is the statute constitutional? (A) No, because it discriminates against the poor in violation of the Equal Protection Clause. (B) No, because it violates the Privileges and Immunities Clause of Article IV, Section 2. (C) Yes, because it is rationally related to a legitimate government interest. (D) Yes, because it violates the Due Process Clause of the Fourteenth Amendment.

(B) No, because it violates the Privileges and Immunities Clause of Article IV, Section 2.The Privileges and Immunities Clause of Article IV, Section 2 protects the fundamental right of citizens to travel freely from state to state. Pursuant to this, durational residency requirements for dispensing government benefits are subject to strict scrutiny. While a reasonable residency requirement would likely be found valid, a one-year waiting period before being able to receive welfare benefits or state-subsidized medical care has been found invalid.

A prominent attorney was suspected of growing marijuana in his home. Based on an anonymous tip, the police went to the lawyer's neighborhood just before dawn. The police could not see into the lawyer's house from the street due to lack of overhead street lights. Seeing that a neighbor's yard looked directly into the lawyer's home, the officers climbed over a fence and stood in the neighbors' yard and looked through the man's living room window. The officers observed what appeared to be marijuana plants being cultivated under grow lights in the kitchen. Based on the tip and their observations, the officers then obtained a search warrant which turned up 40 large marijuana plants. Following the lawyer's arrest, he brings a motion to suppress claiming that the evidence supporting the warrant was obtained through a search that violated the Fourth Amendment. Will the lawyer be successful in his motion to suppress? (A) No, because the house was searched pursuant to a valid warrant. (B) No, because the Fourth Amendment does not protect unlawful activities conducted in plain view. (C) Yes, because the officers' clandestine observation of the plants violated the lawyer's reasonable expectation of privacy concerning activities occurring in his home. (D) Yes, because no unlawful activities could be observed by the officers from any public vantage point.

(B) No, because the Fourth Amendment does not protect unlawful activities conducted in plain view. The marijuana plants were in plain view of the neighbors, and the lawyer has no standing to complain of any police trespass on the neighbors' property [Horton v. California, 496 U.S. 128 (1990)].

On Monday, a chicken farmer emailed a restaurant owner promising to sell him 500 chickens for $2,000. On Tuesday, the owner replied to the farmer that he was rejecting the offer because the price was too high. The farmer quickly replied, "Why don't you wait and think it over?" The following morning, the owner learned that due to a salmonella outbreak, there was a nationwide chicken shortage. He immediately telephoned the farmer to accept. The farmer knew it was the owner, and so he answered the phone by saying, "Good morning, I hereby revoke my offer." The owner replied, "You told me yesterday to think it over, and I have, I accept." The farmer then replied, "You had your chance and you blew it." After the farmer refused to sell the chickens to him, the owner sued for breach of contract. Is there an enforceable contract between the parties? (A) No, because the offer terminated due to a lapse of time. (B) No, because the farmer revoked the offer before the owner accepted it. (C) Yes, because the farmer's statement, "Why don't you think it over until tomorrow," revived the offer, which the owner subsequently accepted in a timely fashion. (D) Yes, because the farmer made the owner a firm offer, which was irrevocable.

(B) No, because the farmer revoked the offer before the owner accepted it. This is a contract for the sale of goods, and so the UCC will govern. There was a valid offer made by the farmer, which was rejected via email by the restaurant owner. There was then a second offer made, or a revival of the original offer, when the farmer told the restaurant owner to wait and think over the decision. Like the first offer, the revived offer was revocable, because it was not a firm offer. Because the farmer revoked the offer before the owner accepted, there was no longer an offer for the owner to accept.

A patient sued his doctor for malpractice. The patient called an expert witness to testify that the drugs prescribed by the doctor were so experimental that it constituted negligence under accepted practices in the medical community. On cross-examination, the expert stated that Pharmacology: A Guide for the Practitioner was a reliable authority in her area of specialty. At trial, it was also stipulated that the treatise in question was first published at the beginning of the last century and had been reprinted every year since with updates. The doctor's attorney then proposed to enter the treatise as an exhibit, highlighting a passage which stated that the drugs prescribed by the doctor are widely used by other physicians in treating patients. The patient's attorney objected. Will the court admit the treatise? (A) No, because the text of the treatise constitutes inadmissible hearsay. (B) No, because the learned treatise exception requires the relevant sections be read to the jury, but not received as evidence itself. (C) Yes, as hearsay falling under the learned treatise exception. (D) Yes, because the jury determines the weight of the evidence.

(B) No, because the learned treatise exception requires the relevant sections be read to the jury, but not received as evidence itself. FRE 803(18) provides that statements contained in treatises may be admitted into evidence during direct or cross-examination of an expert witness if: (1) the treatise is established as a reliable authority; and (2) the treatise is called to the attention of the expert witness during cross-examination or is relied upon by the expert in direct testimony. As an exception to the hearsay rule, statements in treatises admitted pursuant to FRE 803(18) may be used as substantive evidence. They may be read to the jury, but cannot be provided directly to the jury. Therefore, the treatise itself will not be admissible.

Following a judicial determination that it had engaged in past discrimination, a state university agreed to hold 15% of the admissions spots in each department for minority students. A couple years later, a white female student applied to the state university's comparative poetry program and was rejected. The student's aptitude test scores and grade point average were above more than half of the minority students admitted into the program. The student subsequently brought suit against the university claiming that she was unlawfully discriminated against because of her race. Should the court rule in favor of the student? (A) No, because a state may consider race as a factor in admissions when it is attempting to aid disadvantaged minorities and penalizes no particular group. (B) No, because the racial classification is designed to remedy past unlawful discrimination. (C) Yes, because a classification solely by race, even to achieve a worthy purpose, is not necessary to satisfy a compelling state need, and therefore violates the Equal Protection Clause. (D) Yes, because a state university may not use race as a criterion in making admissions decisions.

(B) No, because the racial classification is designed to remedy past unlawful discrimination. While race/ethnic origin may be considered in admissions programs, minority set-asides are not constitutional when they are instituted in an attempt to remedy general societal injustice. However, racial quotas may be used to remedy past unlawful racial discrimination [University of California Regents v. Bakke, 438 U.S. 265 (1978); Grutter v. Bollinger, 539 U.S. 306 (2003)]. Here, the state university has been found guilty of past discrimination, and instituted the affirmative action program under court order. As such, the set-aside here is legal, and the court should rule in favor of the university.

The defendant wanted to steal some documents from his wife's divorce attorney's computer, and so he arranged to have a late-night meeting at the attorney's office. He took with him a small vial filled with a drug used to sedate animals for surgery. The drug was odorless and colorless. When the attorney left the room to take a phone call, the defendant put a small dose of the drug into his drinking cup. After he returned, the attorney took a drink and soon passed out in his chair. The defendant then accessed several documents on the attorney's computer and left. The defendant was unaware that the attorney had severe allergies which were exacerbated by the drug, causing him to suffer a heart attack and die. This jurisdiction defines first-degree murder as the intentional killing of another human being with premeditation, and all other murders are classified as second-degree murder. Manslaughter is defined as at common law. This jurisdiction limits felony murder to inherently dangerous felonies and defines those as at common law. What is the most serious offense of which the defendant can be convicted? (A) First-degree murder. (B) Second-degree murder. (C) Voluntary manslaughter. (D) Involuntary manslaughter.

(B) Second-degree murder. Because the defendant did not have any intent to kill the attorney, the issue becomes one of malice. Malice can be express or implied. Implied malice is measured by the circumstances surrounding the defendant's actions. Here, one could argue that a person lacking sufficient medical training could be malicious in their use of a narcotic. Moreover, while there was no burglary here because this was not a dwelling house, there is a robbery, as the defendant used the narcotic to make the attorney fall unconscious in order to take something from him. The use of fear, force, or intimidation is needed for robbery, and the use of a narcotic would be sufficient to satisfy the force element required. Therefore, the defendant would be guilty of felony murder, which in this jurisdiction would fall within the definition of second-degree murder.

A carpet cleaner sent the following terms in a writing to a homeowner. "I will clean your carpets up to 1,000 square feet for a price of $100 per room. This offer is non-cancellable for 60 days from today." The carpet cleaner dated the letter and signed it at the bottom. The homeowner received the letter and immediately wrote a letter accepting the carpet cleaner's terms, but did not immediately send the letter. A week after sending his letter to the homeowner, the carpet cleaner received an offer from a hotel owner requesting his services for the next month at several of his hotels. The carpet cleaner wanted to accept the hotel owner's offer, but was concerned about the outstanding letter to the homeowner. Which of the following is most accurate? (A) The carpet cleaner can revoke his offer to the homeowner, because the mailbox rule does not apply to option contracts. (B) The carpet cleaner can revoke his offer to the homeowner, because there was no consideration provided that would make the offer irrevocable. (C) The homeowner has manifested his intent to accept, and if he accepts within 60 days of the date of the letter, there is a contract. (D) The homeowner has three months from the date of the letter to accept, because offers from merchants are irrevocable.

(B) The carpet cleaner can revoke his offer to the homeowner, because there was no consideration provided that would make the offer irrevocable. This is an offer for services, rather than goods, and therefore it is governed by the common law. As such, the offer would need to be an option contract, rather than a firm offer, to remain irrevocable. An irrevocable offer could be made if there was consideration offered, but here there was not. Therefore, revocation is effective if it comes before acceptance. If the carpet cleaner revokes his offer, no contract exists with the homeowner.

A defendant was a firm believer that psychics have the power to see into the future. She consulted with a practicing clairvoyant before making any major decisions. The clairvoyant told the defendant that she sensed an "evil presence" in the defendant's life that planned to murder the defendant in the near future. The clairvoyant described the evil presence as someone with red hair who was close to the defendant and whom the defendant would never suspect. The only person with red hair the defendant knew was her sister. The following day, the sister visited the defendant on her way home from work. Believing her sister had come to kill her, the defendant struck her sister in the temple with a fireplace poker, killing her instantly. The defendant was subsequently charged with murder. At her murder trial, the defendant claimed that she killed her sister in self-defense because she believed that her life was threatened. If the jury finds that the defendant honestly believed that she was acting in self-defense, which of the following is true? (A) The defendant will not be convicted of a crime, because she had a good-faith belief that her life was in danger. (B) The defendant will be convicted of voluntary manslaughter because the defendant's belief that her life was threatened was unreasonable. (C) The defendant will be convicted of involuntary manslaughter because the defendant honestly believed that her life was in danger. (D) The defendant will be convicted of murder because the defendant's life was not actually threatened.

(B) The defendant will be convicted of voluntary manslaughter, because the defendant's belief that her life was threatened was unreasonable. This fact pattern presents an example of imperfect self-defense. Imperfect self-defense may mitigate murder to voluntary manslaughter where a defendant was either at fault in starting an altercation, or unreasonably, but honestly, believed that harm was imminent or that deadly force was necessary. In this case, the defendant honestly believed that her sister was going to kill her. However, her belief was unreasonable and thus, her crime will likely be mitigated to voluntary manslaughter.

An emergency room doctor was camping with his family in a local state park. As the doctor was walking to get water, he heard a woman screaming that her husband was choking. The doctor rushed to help, but was unable to dislodge whatever was causing the man to choke, and the man died. The doctor now seeks your advice as to whether he can charge the man's estate for his services. This jurisdiction does not have a Good Samaritan statute, and the hospital that the doctor works at treats all doctors as independent contractors. What advice would be most accurate? (A) The doctor can recover, because he is not a full-time employee at the hospital. (B) The doctor can recover, based on quasi-contract. (C) The doctor cannot recover, because allowing a doctor to benefit in such a situation would be against public policy. (D) The doctor cannot not recover, because the man died.

(B) The doctor can recover, based on quasi-contract. Although there is no formal contract entered into between the man and the doctor, the doctor can recover in quasi-contract for two reasons. First, courts will find an implied-in-law contractual obligation where there is an equitable imposition of a would-be contract. Second, it will prevent unjust enrichment, where one party has bestowed a benefit on the other. Emergency services are a typical situation where courts will find that a quasi-contract exists.

A man offered to buy a woman's farm for $100,000. The woman did not wish to sell her farm, but she jokingly accepted the offer because she did not believe that the man has $100,000. The man and the woman worked out the terms of the contract and the woman, still joking, wrote out the contract on a sheet of paper which both parties signed. The man took the writing and subsequently tried to enforce it, arriving with a check for $100,000. In a suit by the man against the woman, what is the likely outcome? (A) The man will win because they signed the agreement. (B) The man will win, because the contract is binding even if the woman did not intend to sell her farm, as the man actually believed this to be a serious transaction and his belief was reasonable. (C) The woman will win because there was no good faith on her part to enter into a contract. (D) The woman will win because there was no true meeting of the minds.

(B) The man will win, because the contract is binding even if the woman did not intend to sell her farm, as the man actually believed this to be a serious transaction and his belief was reasonable. At the heart of any contract issue is the question of whether or not a contract was actually formed. This usually requires a determination of whether there was valid consideration, and whether there was a valid offer and acceptance. In order for a contract to be formed, there must be mutual assent, which is simply the agreement by both parties to enter into a contract. In deciding whether or not there is mutual assent, courts use an objective "reasonable person" test, in which the court examines the exchange between the parties that led to the establishment of the contract and then determines what a reasonable person in the place of the parties would have understood the exchange to mean. Please note that the court is not interested in what the parties actually thought. It is only interested in what a reasonable person in the same circumstances would have thought. Therefore, even if the woman was joking, if the man actually believed this to be a serious transaction and his belief was reasonable, the court will find that an enforceable contract was validly executed.

On Monday, a man told a gardener, "I am having a party on Sunday and I want my house to look good. If you will agree to mow my lawn by Saturday, I will pay you $50. Think about it and let me know." The man did not hear from the gardener all week. Therefore, the man decided to mow the grass himself. On Saturday, the gardener arrived with his lawn mower, and saw that the grass had been freshly mowed. The man informed the gardener that he had just finished cutting the grass himself. The gardener then brought suit against the man for breach of contract, because he argued that he was within the window of time to accept and that he had given up another job in reliance on mowing the man's grass. Who is likely to prevail? (A) The man, because the gardener never accepted the offer. (B) The man, because the offer to the gardener was indirectly revoked. (C) The gardener, because the man did not specify that acceptance had to be by return promise. (D) The gardener, because he gave up another job in order to accept the man's offer.

(B) The man, because the offer to the gardener was indirectly revoked. Unless specified otherwise, an offer can be accepted by any means reasonable. Here, a return promise was requested, but not received. When the gardener arrived at the house, he became aware that the man had already mowed the grass. At this point, the offer was revoked, and the gardener could no longer accept the offer.

On Monday, a man offered to sell his lawn mower to his neighbor for $100. After receiving the man's offer, the neighbor responded, "Let me think it over." The man then said, "If you say so, but I need to know in a day or two, tops." On Friday, the man sold the lawn mower to his brother. Thereafter, the neighbor decided to accept the man's offer and walked to his house, only to find the man's brother loading it onto his truck. If the neighbor sued the man for breach of contract, judgment should be for whom? (A) The man, because the offer to the neighbor terminated when the neighbor saw the brother loading the lawn mower onto his truck. (B) The man, because the offer was terminated due to the lapse of time. (C) The neighbor, because the offer became irrevocable for a reasonable time when the neighbor asked to "think it over." (D) The neighbor, because the sale of a lawn mower is governed by the UCC.

(B) The man, because the offer was terminated due to the lapse of time. The man agreed to hold the offer open for the neighbor to give the neighbor time to think it over, but the man stated that the offer would only be open for, at most, two days. Thus, when the neighbor failed to accept the offer by the end of Wednesday, two days after the offer was made, the offer lapsed due to time.

A woman had known a hairstylist for many years. One day, the hairstylist was walking through the mall when he ran into the woman. The hairstylist said to the woman, "I will straighten your hair tomorrow for $20. Come by the shop." The woman replied, "That's a great price." Thereupon the woman's husband approached and the three then started gossiping about a man who walked by wearing some out-of-fashion shoes. The next day, the woman arrived at the hairstylist's shop and said, "I accept your offer." The hairstylist replied, "I can't straighten your hair because after we spoke yesterday, I contracted with several other people to do their hair, and now I'm all booked up." If the woman sues the hairstylist for breach of contract, who will likely prevail? (A) The woman, because she relied on the hairstylist's promise by coming to the shop. (B) The woman because the hairstylist made a unilateral offer, and the woman arrived at the shop ready to pay $20 for the haircut. (C) The hairstylist, because he contracted with other people to do their hair. (D) The hairstylist, because the woman did not accept during their conversation.

(B) The woman because the hairstylist made a unilateral offer, and the woman arrived at the shop ready to pay $20 for the haircut. The hairstylist made an offer. While it is true that the woman did not accept the offer during the conversation, the offer was for a unilateral agreement, which the woman accepted by coming by the shop the next day.

A landlord agreed to lease his townhouse to a tenant for a period of one year. The written lease agreement stated that the lease would automatically renew for successive one-year periods unless either the landlord or the tenant gave notice of termination at least one month (and no more than two months) in advance of the end of the period. One year passed after the landlord and tenant executed the lease agreement, and neither gave notice of termination. After the one-year anniversary of the execution of the lease agreement, what type of tenancy do the landlord and tenant have? (A) An implied tenancy. (B) A term of years. (C) A periodic tenancy. A periodic tenancy continues from period to period without a set termination date until proper notice is given. The lease in this example is periodic, because it renews for successive one-year periods until proper notice of termination is given by either party. (D) An at-will tenancy.

(C) A periodic tenancy. A periodic tenancy continues from period to period without a set termination date until proper notice is given. The lease in this example is periodic, because it renews for successive one-year periods until proper notice of termination is given by either party.

While driving her company vehicle near a pedestrian mall, a woman came upon the scene of a three-car accident. She was so busy gawking at the damaged vehicles that she failed to see one of the victims lying on the road in front of her car. She hit and ran over the victim, who survived and sued the woman's company. The victim offers the testimony of a witness to the incident. Referring to the woman, the witness stated, "The driver of that car ran over the victim as he was lying on the ground awaiting an ambulance, and said 'It is all my fault, I should have been paying more attention to my driving.'" Assume for this question that the woman is available to testify. How should the trial judge rule on the admission of the testimony? (A) Admissible, as a statement against interest. (B) Admissible, as a present sense impression. (C) Admissible, as a statement by a party-opponent. (D) Inadmissible, as hearsay not within any recognized exception.

(C) Admissible, as a statement by a party-opponent. A statement will not be barred from admission by the general rule against hearsay where the statement is offered against an opposing party and was made by the party's agent or employee on a matter within the scope of that relationship and while it existed. Indeed, there is a substantial trend favoring the admission of statements relating to matters within the scope of the agency or employment. Therefore, this answer is correct, because the statement will be admitted as a vicarious statement by a party-opponent.

A man and a woman were involved in a car accident that occurred when the man abruptly switched lanes in front of the woman without signaling. Unable to stop, the woman hit the man's car into a median. Immediately after the accident, as the two drivers emerged from their vehicles, a pedestrian, obviously emotional and upset from having just observed an accident, ran up to the man and shouted, "You careless driver, you! Didn't you ever learn that you're supposed to use a turn signal when you switch lanes?" The pedestrian was never identified and is not present at trial. How should the court rule on the pedestrian's statement? (A) Admissible, as non-hearsay. (B) Admissible, even though it is hearsay. (C) Admissible, under the excited utterance exception. (D) Inadmissible, because the bystander cannot be identified and is not present at trial.

(C) Admissible, under the excited utterance exception. Under FRE 803(2), a statement relating to a startling event or condition made while the declarant was under the stress of excitement caused by the event or condition is not excluded by the hearsay rule. The theory behind the excited utterance exception is simply that circumstances may produce a condition of excitement that temporarily stills the capacity of reflection and, thus, produces utterances free of conscious fabrication. Spontaneity is the key factor in determining whether the statement was, indeed, made without any conscious reflection.

The defendant was driving down the highway, which had a posted speed limit of 65 miles per hour, at 90 miles per hour when a police officer pulled him over for speeding. As per protocol, the officer called for back-up. The closest officer, who responded to the scene, had a drug-sniffing dog in his car. When the back-up officer arrived, they asked the defendant to step out of the car. While the first officer was questioning the defendant, the second officer allowed his dog to sniff the defendant's car. The dog quickly alerted the officer that he smelled drugs in the trunk of the defendant's car. Which of the following is true? (A) Allowing the dog to sniff the exterior of the car constitutes a search of the vehicle, subject to Fourth Amendment protections. (B) Police may only use a drug-sniffing dog when they already have probable cause to search the defendant's vehicle. (C) Allowing the dog to sniff the exterior of the car does not violate the privacy rights of the suspect. (D) Allowing the dog to sniff the exterior of the car in this situation violates the privacy rights of the suspect.

(C) Allowing the dog to sniff the exterior of the car does not violate the privacy rights of the suspect. The automobile exception is based on the idea that a lesser expectation of privacy exists in an automobile, boat, or airplane than in one's home or personal property. Moreover, the use of drug-sniffing dogs trained to detect contraband but not other items does not invade the privacy rights of the suspect. Therefore, in this case, the defendant's privacy rights in his vehicle were not violated when the police allowed the drug-sniffing dog to sniff his car. Thus, this is the correct answer choice.

A chef negotiated with a contractor to remodel his home kitchen to look like the kitchen in his restaurant. In order to do this, the contractor had to order top-of-the-line appliances. He also needed to remove some sections of a wall and install new electrical sockets to accommodate the appliances. The cost of the remodel was $50,000, with two-thirds of the price coming from the purchase of the appliances. A dispute arose and the parties both suspended their obligations. The contractor refused to further perform and the chef refused to pay the contractor. What law should be used to analyze the dispute? (A) The UCC applies to the purchase of the appliances and common law applies to the delivery, the wall removal, and the electrical socket installation. (B) The UCC should be applied to the entire transaction, because the primary purpose of the contract was the appliances, which account for the overwhelming majority of the contract price. (C) Common law should be applied to the entire transaction, because the primary purpose was to provide a service, regardless of the cost of the appliances. (D) When a contract is for both goods and services, in the absence of a choice-of-law selection clause, common law is the default law to be applied.

(C) Common law should be applied to the entire transaction, because the primary purpose was to provide a service, regardless of the cost of the appliances. Following the predominant purpose test, a court will view the predominant purpose of the contract to be for services--specifically, a kitchen remodel. Where a contract has a mix of goods and services, relevant criteria for determining whether the UCC will control will include: (1) the contract language; (2) the nature of the seller's business; (3) the reason for entering the contract; and (4) the amounts charged under the contract for the goods and services. Here, although most of the contract price was for the purchase of the appliances, the purchase was not made directly to an appliance store, where delivery was the only service. Rather, the chef went to a contractor, presumably for his skill and experience in remodeling. The end result that the chef was contracting for was not the appliances, but a kitchen that looked like the one he used in his restaurant. As such, this is primarily a services contract, with the goods being incidental to the agreement.

On Monday, Jim decided to sell his vintage car. Bill had always admired the car and inquired as soon as he saw the for-sale sign. Jim said he was firm at $5,000. Bill thought this was a good price, and told Jim he wanted it, but needed to check with his wife before spending such a large amount of money. In the meantime, Bill pulled a $100 bill from his pocket and gave it to Jim, saying, "Here's a down payment. See you tomorrow!" Jim replied, "Okay, it will be here." On Tuesday morning, Bill approached Jim with a check written out for $4,900. Jim said, "Sorry, but you're too late. I accepted an offer last night for $10,000." In an action by Bill against Jim for breach of contract, how should the court rule? (A) For Jim, because $100 is insufficient to create an option contract for a $5,000 car. (B) For Jim, because Bill's language was insufficient to create a valid option contract. (C) For Bill, because Jim created an option contract, which is irrevocable at least until the following day. (D) For Bill, because the UCC precludes Jim from selling the car to anyone other than Bill before Wednesday.

(C) For Bill, because Jim created an option contract, which is irrevocable at least until the following day. An option contract is a promise to keep an offer open for a period of time and which is supported by consideration. With an option contact, the offeror is not permitted to revoke the offer during the stated period, because with the payment, he is bargaining away his right to revoke the offer. Because Bill paid Jim $100 to keep the offer open until the next day, the offer was still open when Bill then attempted to accept it.

A baseball fanatic became enraged when a fan for the other team reached out into the field of play and touched a ball, causing the winning run to score. As the spectators were leaving the stadium, the fanatic saw the other fan, who had touched the ball, walking to his car. The fanatic walked up behind the other fan and punched him in the back of the head. The other fan, who had been drinking all day, was immediately knocked out and died as soon as his head hit the pavement. This jurisdiction defines assault as: (1) the causing of physical harm to another (categorized as felony assault); or (2) acting in a threatening manner to put another in fear of immediate harm (categorized as misdemeanor assault). The fanatic was arrested for the homicide of the other fan. Which of the following is the most serious crime for which the fanatic should be found guilty? (A) Assault. (B) Voluntary manslaughter. (C) Involuntary manslaughter (D) Felony murder.

(C) Involuntary manslaughter. As a general rule, whenever an intentional battery or assault results in an unintended death, the defendant is guilty of involuntary manslaughter. This is a good example of misdemeanor manslaughter.

In the latest of a long series of government corruption scandals, a state employee was convicted in federal court of taking kickbacks in exchange for influencing government contract decisions. The employee was sentenced to time served plus probation, fined heavily, and given a significant amount of community service. The employee resigned from his position in the government and began to collect his pension, for which he had fully qualified after spending several decades in government service. The media began to lash out against the state government for what it viewed as lax punishment by a corrupt government. In response, the state legislature passed, and the governor signed into law, a bill which stripped former government employees of any retirement or other benefits or payments of any kind from the state if the employee was convicted of bribery or corruption, declaring that such a conviction amounted to breach of the government official's employment contract. As a result of the new law, the state employee was notified that the state was immediately discontinuing pension benefit payments to him on account of his conviction. The state employee then properly filed a lawsuit challenging the termination of his retirement benefits on the grounds that the new law was unconstitutional. Which of the following is the best argument the state could make in favor of the law's constitutionality? (A) The notification that the state employee received was sufficient notice to satisfy due process for the discontinuation of pension benefits. (B) Deprivation of pension benefits is not cruel and unusual punishment. (C) It is implicit that one of the conditions of the state's contract of employment with a government official is that he shall not take kickbacks or otherwise engage in bribery. (D) The state employee was afforded an opportunity to express his views about the new legislation at public hearings prior to the enactment of the statute.

(C) It is implicit that one of the conditions of the state's contract of employment with a government official is that he shall not take kickbacks or otherwise engage in bribery. This question requires a 2-step analysis to select the best alternative. First, you must determine the constitutional issue involved. 2nd, you must apply the appropriate constitutional principle to the question asked--i.e., the state's best argument against a constitutional challenge to the forfeiture statute. This answer choice is correct because the argument concerning a condition of employment contract with a government official affects the Contract Clause of the Constitution. The state employee could validly challenge the constitutionality of the statute, alleging unconstitutional impairment of the obligation to contract. The pension forfeiture statute would be violative of the Contract Clause because under the circumstances, the state employee has satisfied the conditions of retirement eligibility (he was fully qualified for his pension, as stated in the facts). His retirement pay has ripened into a full contractual obligation and has become a vested right. Therefore, the pension forfeiture statute would be an unconstitutional impairment of his vested right to receive retirement benefits (his pension).

In order to combat a rise in organized crime within the state, an anti-organized crime task force was created. After reviewing the previous attempts by various state and local agencies to infiltrate the criminal organizations, the state also adopted a policy that prohibited the assignment of female agents to the undercover infiltration division of the task force. Over the course of 10 separate attempts, seven agents were killed attempting to go undercover, including all five women that were sent in, none of whom managed to make even an initial penetration into the crime families. While undercover assignments were a very successful way to advance to a higher position, it was because of the state's concern with the safety and well-being of its female officers and agents and the demonstrated misogyny of the crime families under investigation that it adopted such a policy. One female agent, desiring to be a member of the undercover division in order to help her achieve her next promotion, submitted an application for the undercover division. After her application was rejected, the agent sued the state in federal court to enjoin enforcement of its stated policy on the grounds that it was unconstitutional. As a matter of constitutional law, which of the following results in this suit is most appropriate? (A) Judgment for the agent, because the particular classification contained in this policy is not necessary to further a compelling interest. (B) Judgment for the agent, because the terms and conditions of state government employees are privileges or immunities of state citizenship that may not be abridged by the state on the basis of gender. (C) Judgment for the state, because the policy is substantially related to the advancement of an important state interest. (D) Judgment for the state, because the state has articulated a rational basis for this classification and, therefore, a court may not substitute its judgment for that of responsible state officials.

(C) Judgment for the state, because the policy is substantially related to the advancement of an important state interest. Gender classifications are subject to intermediate scrutiny, which puts the burden of persuasion on the government to demonstrate that the classification (or discrimination) is substantially related to an important government interest. Here, there is a record showing that such an assignment carries a very high risk of death for female agents, due in large part to the nature of the groups being infiltrated. As such, this classification will likely be found to substantially relate to an important state interest.

A heartsick lover knew he was being foolish, but he just could not stop himself. The woman he was in love with was so beautiful, and she was so lonely with her husband in prison. She had told the lover that her husband was extremely jealous. She said that her husband was in prison because he had shot her last lover when he learned of their affair. About four months after he started seeing the woman, the lover received a telephone call from her. She seemed distraught, and cried, "My husband escaped from prison this evening, and he knows about us!" As expected, the husband called on the lover that evening. The lover shot the husband and was prosecuted for his murder. The lover claims self-defense. At trial, he attempts to testify to the woman's statement that her husband was in prison because he had shot her last lover when he learned of the affair. If the statement is offered to prove that the husband was in prison because he had shot the woman's lover when he learned of the affair, should the statement be admitted? (A) Yes, under the state-of-mind exception. (B) Yes, as nonhearsay. (C) No, as hearsay not within any exception. (D) No, because it is self-serving.

(C) No, as hearsay not within any exception. Under FRE 801(c), hearsay is: (1) any statement other than a statement made by the declarant while testifying at the trial or hearing in which the statement is proffered; (2) which is offered into evidence in order to prove the truth of the matter asserted. Here, this is hearsay because the question tells us that the lover offers the statement for the express purpose of proving the truth of the matter asserted, i.e., that the husband had killed his wife's prior lover. The statement does not fall within any exception, thus it is not admissible.

Police received a tip from an informant that a ring of thieves were hoarding stolen property in the house owned by the ringleader's uncle. The ringleader frequented the uncle's house to sell the stolen goods out of the basement and could be found there every evening after 10:00 pm. The police obtained an arrest warrant and went to the uncle's house to arrest the ringleader the next evening. The uncle refused entry to the police, who pushed past him to find the ringleader. When police entered the living room, they observed bricks of marijuana on the table. Police arrested the uncle and the ringleader and charged them both with narcotics offenses. The uncle's motion to suppress the evidence of the drugs was granted. The ringleader files a similar motion seeking suppression of the drugs. Should the ringleader's motion be granted? (A) Yes, because the police did not have a valid search warrant for the uncle's house. (B) Yes, because there was no probable cause to believe that the drugs belonged to the ringleader. (C) No, because the ringleader had no standing to contest the legality of the search. (D) No, because the drugs were observed in plain view and there were exigent circumstances.

(C) No, because the ringleader had no standing to contest the legality of the search. Standing under the 4th Amendment is personal to the defendant. Thus, in order for the ringleader to have standing to vindicate his own rights under the 4th Amendment, he must have a reasonable and legitimate expectation of privacy in the place searched or the item seized. The ringleader does not have a legitimate expectation of privacy in someone else's home and therefore, does not have standing.

A police officer sees a robber point a pistol at a couple and take items from them before fleeing on foot. The officer chases the robber on foot. The robber, with gun still in hand, runs into his nearby apartment. This apartment is being leased by the robber. The officer follows the robber into the apartment and the officer realizes that there is no rear exit. The officer begins to search the apartment in an attempt to locate the robber. When the officer opens a hallway closet door, he sees a large quantity of drugs on the floor. The officer then finds the robber under the bed in the master bedroom. The officer arrests the robber for the armed robbery and possession of drugs with the intent to distribute. At a pre-trial motion, the robber's attorney argues that the drugs are inadmissible evidence because the officer did not have a warrant to search the property. How will the court rule? (A) The court should suppress the evidence, because the officer needed a warrant to enter the apartment. (B) The court should suppress the evidence, because the drugs were not in plain view. (C) The court should admit the evidence, because the officer was in hot pursuit. (D) The court should admit the evidence, because there were exigent circumstances.

(C) The court should admit the evidence, because the officer was in hot pursuit. Police may enter & search a private dwelling while in reasonable pursuit of a fleeing suspect. In such a circumstance, what would otherwise have been a warrantless search becomes lawful because the police are in actual "hot pursuit" of a fleeing suspect and actively attempting to apprehend him. While in that pursuit, they may seize not only evidence of the crime for which they are chasing the suspect but also any additional contraband that they find. Here, the officer was in hot pursuit of the robber and while attempting to apprehend him, the officer chased the robber into an apartment. The drugs were located in a closet while the officer was looking for the robber. Therefore, the drugs were lawfully seized contraband and can be admitted as evidence.

A man and his friend decided to rob a bank. They entered the bank and quickly got into a shootout with the security guard. While the man was taking the money from the vault, the security guard was struck by a bullet and killed. The two men then left and, as the man was following his friend to the car, the man shot the friend in the back. This jurisdiction defines all murders as second-degree murder unless deliberation and premeditation can be shown, in which case the crime is elevated to first-degree murder. Manslaughter is defined as at common law. This jurisdiction also follows the agency theory for felony murder. Which of the following statements is most accurate? (A) The defendant is guilty of first-degree murder of the security guard and second-degree murder of the friend under the agency theory. (B) The defendant is guilty of first-degree murder of security guard and the friend under the agency theory. (C) The defendant is guilty of second-degree murder of the security guard and first-degree murder of the friend. (D) The defendant is guilty of second-degree murder of both the security guard and the friend, because both were killed during the felony, and neither death was premeditated.

(C) The defendant is guilty of second-degree murder of the security guard and first-degree murder of the friend. In this jurisdiction, murder is divided into two degrees for the purpose of imposing a more severe penalty for some murders than for others. First-degree murder includes intent-to-kill murder accompanied by premeditation and deliberation. Murder not falling within the definition of first-degree murder is considered second-degree murder. Here, the defendant's act toward the security guard is best described as felony murder. Even though he did not kill the guard (his friend did), under the agency theory of felony murder, a felon is responsible for the actions of his co-felons. Because the guard was shot by the friend, the defendant will be responsible. That killing was committed during the course of the armed robbery, and therefore, it will be felony murder, which is second-degree murder in this jurisdiction. Additionally, because the man intentionally shot his friend, that will be first-degree murder in this jurisdiction. Premeditation can occur in a moment, and here, the shooting was from behind, evidencing an intent to kill.

A police officer sees a robber point a pistol at a couple and takes items from them before fleeing on foot. The officer chases after the robber. The robber, with the gun still in hand, runs into a nearby apartment. This apartment is being leased by the robber. The officer follows the robber into the apartment and soon realizes that there is no rear exit. The officer begins to search the apartment in an attempt to locate the robber and finds a large number of drugs when he opens a hallway closet. The robber is eventually located under a bed in another room. The officer arrests the robber for the armed robbery and possession of drugs with the intent to distribute. At a pretrial motion, the robber's attorney argues that the drugs are inadmissible evidence because the officer did not have a warrant to search the property. How will the court rule? (A) The drugs are inadmissible because the officer needed the warrant to enter the apartment. (B) The drugs are inadmissible because the drugs were not in plain view. (C) The drugs are admissible because the officer was in hot pursuit. (D) The drugs are admissible because there were exigent circumstances.

(C) The drugs are admissible, because the officer was in hot pursuit. Police may enter and search a private dwelling while in reasonable pursuit of a fleeing suspect. In such a circumstance, what would otherwise have been a warrantless search becomes lawful because the police are in actual "hot pursuit" of a fleeing suspect and actively attempting to apprehend him. While in that pursuit, they may seize not only evidence of the crime for which they are chasing the suspect but also any additional contraband that they find. Here, the officer was in hot pursuit of the robber and while attempting to apprehend him, the officer chased the robber into an apartment. The drugs were located in a closet while the officer was looking for the robber. Therefore, the drugs were lawfully seized contraband and can be admitted as evidence.

A six-year-old girl was kidnapped outside her home in a small town. Following the girl's abduction, her parents publicly announced a $50,000 reward to anyone responsible for the apprehension of the kidnapper. The girl's abductor was described as a Caucasian male between the ages of 25 and 30 with curly blond hair. He also was identified as having a pentagram tattoo on his left arm. One afternoon, an off-duty police officer from a nearby town stopped at a local fast-food restaurant for lunch. The officer was aware of the kidnapping but was unaware of the reward. Once he entered the restaurant, the police officer noticed a man fitting the description of the kidnapper. The police officer proceeded to arrest the individual, who, in fact, turned out to be the girl's abductor. After the girl was found unharmed, the police officer learned of and requested the $50,000 reward. However, the girl's parents refused to pay the police officer the reward money. If the police officer brings suit against the girl's parents to recover the $50,000 reward, for whom should the court enter judgment? (A) The police officer, because he accepted the offer by apprehending the kidnapper. (B) The police officer, because he was off duty when the apprehension occurred. (C) The girl's parents, because the officer was unaware of the reward when he apprehended the kidnapper. (D) The girl's parents, because recovery of a reward by a police officer is against public policy.

(C) The girl's parents, because the officer was unaware of the reward when he apprehended the kidnapper. This question deals with the preexisting duty rule. In this question, the issue is whether an offeree can accept a reward that they were unaware was offered before they acted. The answer is no, if the requested action occurs before the offeree is aware. Because the officer was unaware of the reward offered at the time that the apprehended the kidnapper, he may not obtain it later.

A state legislature, in an attempt to curb teen pregnancies, recently enacted a statute denying welfare assistance to parents for illegitimate children. A young woman was receiving welfare assistance after being awarded sole custody of her two children following her divorce several years earlier. She recently gave birth to a third child and applied for an increase in her welfare assistance as a result. However, her request was denied pursuant to the state law. The woman sued the state, claiming that the law was unconstitutional and violated the Equal Protection Clause. Which of the following correctly states the burden of persuasion in this case? (A) The woman has the burden of persuading the court that the statute is not necessary to further a compelling interest. (B) The woman has the burden of persuading the court that the statute is not substantially related to achieve an important government interest. (C) The state has the burden of persuading the court that the statute is substantially related to achieving an important government interest. (D) The state has the burden of persuading the court that the statute is rationally related to a legitimate government interest.

(C) The state has the burden of persuading the court that the statute is substantially related to achieving an important government interest. Intermediate scrutiny applies to government action using quasi-suspect classifications, such as gender and illegitimacy. Here, the statute treats legitimate and illegitimate children differently. As such, it will be subjected to the intermediate scrutiny standard. Under this standard, the burden is on the state to prove that the measure being challenged is substantially related to the achievement of an important government interest.

A woman was on trial for the murder of her husband. The prosecution alleged that the woman paid one of her neighbors $25,000 to kill her husband. Immediately after the killing, the neighbor boarded a bus and called his roommate. In a lowered voice, he whispered, "Pack your bags, we gotta get out of town for a while 'till the heat cools off. Don't worry about money, I got it covered." The neighbor's conversation was heard by a fellow passenger. The neighbor was arrested at the airport the following day. At trial, the prosecution attempted to call the neighbor to testify, but he invoked his Fifth Amendment right not to testify. The prosecution then called the passenger to testify. The wife objected. Which of the following is most accurate? (A) The statement is inadmissible because it does not directly incriminate the neighbor. (B) The statement is inadmissible because it does not directly incriminate the wife. (C) The statement is admissible, as a statement against interest. (D) The statement is admissible, as an excited utterance.

(C) The statement is admissible, as a statement against interest. If a declarant makes a statement against his penal interest and is unavailable for trial, the statement will qualify as a statement against interest and will be admitted as an exception to the hearsay rule. The facts indicate that the neighbor has invoked his Fifth Amendment right not to testify. The neighbor's statement also meets this exception's criteria that it was against the neighbor's penal interest when he said it, and that a reasonable person would not have made the statement if it were not true. Therefore, this is the best answer.

A man was a convicted of sexual assault and required to register with the county of his residence within 30 days of his release from prison. After being paroled, the man rented an apartment, but failed to register. Three months later, during a routine traffic stop, the man was arrested for failing to register in violation of his parole. At trial, the prosecutor called the chief clerk of the county in which the defendant's apartment was located to testify. She proposed to testify that she did a diligent search of the country sex offense registry and that the defendant's name did not appear in the records. The defendant objected, claiming that her testimony was hearsay. How should the court rule? (A) The testimony is inadmissible, under the Confrontation Clause. (B) The testimony is inadmissible because the business record exception applies only to an actual entry in a business record. (C) The testimony is admissible, as proof of the absence of an entry in a business record. (D) The testimony is admissible because the clerk has firsthand knowledge of the country records.

(C) The testimony is admissible, as proof of the absence of an entry in a business record. In order to prove who has not registered as a sex offender, the clerk must testify to a document that lists all of the registered sex offenders. Because this list is an out-of-court statement being offered for its truth, a hearsay exception must exist for it to be admissible. Here, the prosecution is attempting to prove that the man is not on the list, and therefore not compliant with the law. As such, the absence of an entry in a business record is what is being proved. The elements are similar to the business record exception; however, it is being used to show what is not there, rather than what is there. Because the man's absence from the list is what is being sought to be proved, this exception provides for the clerk's testimony.

A woman who was missing one of her legs applied to be a city firefighter, but was rejected. The woman then contacted the station chief, who admitted that the woman was denied admission solely due to the fact that she was missing one leg. The woman then filed an appropriate action against the city asserting that her equal protection rights had been violated. Which of the following correctly states the applicable burden of persuasion? (A) The city has the burden to show that the woman's rejection furthers an important state interest. (B) The city has the burden to show that the woman's rejection is rationally related to a legitimate state interest. (C) The woman bears the burden to show the city's actions are not rationally related to a legitimate state interest. (D) The woman bears the burden to show the city's actions do not further a compelling state interest.

(C) The woman bears the burden to show the city's actions are not rationally related to a legitimate state interest. Discrimination based on a physical disability, such as a missing leg, implicates neither a fundamental right nor a suspect class. As such, rational basis scrutiny would be applied. The burden of persuasion would be on the plaintiff, the woman, to show that the city's actions are not rationally related to a legitimate state interest. Notwithstanding certain protections created by state governments and Congress (notably, the ADA) to prevent discrimination against persons with disabilities, the fact that the woman was rejected based on his physical disability per se does not warrant the use of heightened equal protection scrutiny.

A rare coin dealer and a collector met at a stamp-and-coin fair that was held on a monthly basis. The dealer had a mint condition 1905 Silver Eagle in his inventory that was valued at well over $10,000. The collector desperately wanted to purchase the coin, but knew that he did not have enough money to make the purchase. The two spoke for a while, and before leaving, the following note was written from the dealer to the collector: "I will hold the 1905 Silver Eagle you were eyeing up at the fair today for sale to you alone until the next fair." The dealer signed and dated the note and handed it to the collector. If the collector wanted to purchase the coin at the following fair, is the offer still in force? (A) No, because there was no consideration provided to keep the offer open until the next fair. (B) No, because "until the next fair" does not state a sufficient period of time. (C) Yes, because it was a firm offer. (D) Yes, because the offer concerned the sale of goods.

(C) Yes, because it was a firm offer. This question concerns a sale of goods by a merchant. The writing from the dealer to the collector here constitutes a firm offer as defined by the UCC. A written offer made by a merchant that expressly states that it will be held open is effective and irrevocable for the stated period (up to a maximum of three months).

A state agency needed to select an officer to go undercover and infiltrate a white supremacist group whose members occupied a fortified compound within the state. A Hispanic officer with extensive undercover experience applied for the assignment but was rejected in favor of a much less experienced white officer. The agency admitted that race was the determinative criteria in filling the position. If the Hispanic officer sued, should the court uphold the department's decision? (A) No, because it was a violation of the Equal Protection Clause of the 14th Amendment. (B) No, because race cannot be used as a criterion in government job assignments. (C) Yes, because it was necessary to achieve a compelling interest. (D) Yes, because it was rationally related to a legitimate state interest.

(C) Yes, because it was necessary to achieve a compelling interest. The primary criterion for the government job assignment in this case is race. Under the Equal Protection Clause, that classification is only valid when necessary to achieve a compelling interest. This is one of the rare cases in which this standard is met. A Hispanic undercover agent would be totally ineffective penetrating an organization in which race is the primary criterion for membership. Therefore, the racial classification in this case will be upheld.

Knowing that his niece was looking at a red sedan on his used car lot, an uncle sent a signed email to his niece one Friday afternoon, promising to sell her the red sedan from his lot for $1,000, and stating that she could have the weekend to think about it and let him know. After reading the email, the niece sent her uncle a reply that stated, "The price is a little high. I'll give you $800." The uncle responded via email, "I will not accept $800; it is worth more than that." On Saturday, the niece emailed her uncle and said, "I changed my mind, I will pay $1,000." On Sunday, the uncle responded that he refused to sell the niece his car. If the niece brings a cause of action against her uncle to enforce the agreement, will she prevail? (A) No, because a firm offer is immediately revoked with a rejection. (B) No, because the niece did not provide any consideration to create an option contract. (C) Yes, because she accepted the uncle's terms before the uncle took any action contrary to the offer to the niece. (D) Yes, because the uncle was a merchant

(C) Yes, because she accepted the uncle's terms before the uncle took any action contrary to the offer to the niece. Not all statements or questions about an offer are considered counteroffers. This was a firm offer, because the uncle was a merchant and stated in a signed writing that he would hold the offer open for a specified period of time. As such, the offer was irrevocable during this period. When there is an irrevocable offer (either a firm offer under the UCC or an option contract at common law), the offer remains open even in the face of an outright rejection by the offeree. However, the offer will be revoked if the offeree rejects it and the offeror, in the face of the rejection, changes his position in reliance on the rejection. If the offeror has not changed his position in reliance, the offeree may still accept the offer during the period even after previously rejecting it. Therefore, the niece could still accept the uncle's offer here despite having rejected it by making a counteroffer, because the uncle has not changed his position in reliance on the niece's rejection.

A businessman lived in Japan. He sent a letter to a California car dealership stating that if the dealership would extend his son the credit, he would guarantee the purchase price of any car. The dealership received the letter on February 9, and on February 10, sold a car to the son for $11,500. On February 11, the businessman died suddenly. Unaware of his death, the agency sent him a letter indicating its acceptance of the offer and notifying him of the sale. The dealership also noted that the son had a good credit rating and that the dealership would have extended him credit even without his father's guarantee. Two months later, the son also died suddenly, leaving a bankrupt estate. The dealership then sought to recover the balance of the car's purchase price from the businessman's estate. Would the dealership prevail in a suit against the businessman's estate for the balance owed on the car? (A) No, because the dealership would have sold the car to the son even without the businessman's guarantee of the purchase price. (B) No, because the businessman died before the dealership mailed the letter notifying him of its acceptance. (C) Yes, because the dealership accepted the businessman's offer before his death. (D) Yes, because the dealership foreseeably, justifiably, and reasonably relied on the businessman's promise.

(C) Yes, because the dealership accepted the businessman's offer before his death. A unilateral contract is created when the offeror requests acceptance of the offer by the performance of an act rather than by a promise to perform the act. The offer is accepted only when the offeree performs the requested act. An offer is revoked by operation of law upon the death of the offeror. Here, given that the businessman's offer requested acceptance by performance rather than by a promise to perform, the dealership accepted the offer when it performed the act that the businessman had requested (i.e., extending his son credit). Therefore, the businessman's offer created a unilateral contract, which can be enforced against his estate. The fact that the dealership notified the businessman of its acceptance of the offer after his death is not the controlling factor; what is controlling is that by performing the requested act, the dealership validly accepted the offer (by its performance of the requested act) before the businessman died.

A group of women having a bachelorette party were inside of a club when a shooting occurred behind their table. The bartender was arrested for aggravated assault for the shooting. At trial, the bride-to-be took the stand to testify. She admitted that she did not actually see the shooting but that she heard someone behind her yell, "Watch out! He's got a gun! The bartender's got a gun!" She testified that she heard this being yelled at the same time that she heard the gunshots. There was only one bartender, a male, employed by the bar that night. The person yelling never came forward to speak to the police about the shooting. Can the bride-to-be testify about what she heard? (A) No, because the declarant's statement it is hearsay not within any exception. (B) No, because the declarant is unavailable. (C) Yes, because the statement describes an event while the declarant was perceiving that event. (D) Yes, because there is sufficient corroboration presented, therefore this is a question of weight, not admissibility.

(C) Yes, because the statement describes an event while the declarant was perceiving that event. This answer is the most accurate because it presents a portion of the definition of the present sense impression exception to the hearsay rule. With this exception, a statement which describes or explains an event or condition made while the declarant was perceiving the event or condition, or immediately thereafter, are not excluded by the hearsay rule whether the declarant is available to testify or not. Here, the unidentified declarant yelled out statements regarding the shooter in a club. The statements essentially identified the shooter. The statements were made contemporaneous to the shooting and while the declarant perceived the shooting. Therefore, the Present Sense Impression exception would apply and allow the bride-to-be to testify about the statements in order to prove the truth of the matter asserted.

A reliable police informant telephoned the police to report seeing two men in a blue car selling narcotics outside a local movie theater. The informant gave a detailed description of the two men and the license number of the vehicle. A patrol car was immediately dispatched to the movie theater. A few minutes later, the police saw a car matching the description given parked about two blocks from the movie theater. When the police approached, they saw one man fitting the description provided. The police proceeded to the car and opened the doors of the vehicle. The police then pried open a locked tool case that was situated in the rear of the vehicle. Inside, the police found an envelope that contained about one gram of cocaine. They then placed the driver of the vehicle under arrest. Immediately thereafter, the police opened the glove compartment and found a small amount of marijuana. The driver of the van was subsequently prosecuted and charged with two counts of possession of controlled dangerous substances. He filed a motion to exclude the cocaine and marijuana from evidence. How will the court most likely rule on his motion? (A) Granted, as to the cocaine only. (B) Granted, as to the marijuana only. (C) Granted, as to both the cocaine and the marijuana. (D) Denied, as to the cocaine and the marijuana.

(D) Denied, as to the cocaine and the marijuana. In California v. Acevedo [500 U.S. 565 (1991)], the Supreme Court held that police may search an automobile and the containers within where they have probable cause to believe contraband or evidence is contained. Accordingly, in United States v. Ross [456 U.S. 798 (1982)], the Court held that if "probable cause justifies the search of a lawfully stopped vehicle, it justifies the search of every part of the vehicle and its contents that may conceal the object of the search." Since the police had probable cause to search the entire vehicle for drugs, the police may lawfully search both the tool case and the glove compartment.

A defendant, who is divorced, lives with her 11-year-old daughter. One night, the defendant went out to dinner and left her daughter at home. Upon leaving the house, the defendant locked the door to her bedroom. While the defendant was gone, the daughter found a spare key to her mother's bedroom and opened the door. While rummaging through her mother's dresser, she found a bag containing white powder. Concerned that the bag might contain drugs, the daughter called her father, who still owned the home along with the defendant, to ask his advice. The father instructed her to call the police. The daughter then telephoned the police station and reported what she had found. A patrol car was immediately dispatched to her home. The daughter allowed the police to come inside the home. The officers then asked her where the bag that she described to the 911 operator was located, and if she could take them to it. The daughter led the officers to the mother's bedroom and gave them the bag, which they took to the station house for a lab test. After the results confirmed that the bag contained cocaine, the defendant was arrested and charged with illegal possession of a controlled dangerous substance. The defendant's attorney filed a motion to prevent the cocaine from being admitted into evidence. How should the court rule on the motion? (A) Denied, because the daughter, and her father, as co-owner, had the authority to allow the police inside the home and to make the seizure. (B) Denied, because the daughter lived with her mother, and she had equal control over the premises. (C) Granted, because the daughter, as a minor, did not have the apparent authority to permit the police to enter the home. (D) Granted, because the daughter did not have the apparent authority to permit the police to enter the defendant's bedroom.

(D) Granted, because the daughter did not have the apparent authority to permit the police to enter the defendant's bedroom. The consent search exception to the 4th Amendment warrant requirement is being tested here. Any person with an apparent equal right to use or occupy the property may consent to a search, and any evidence found may be used against the occupants [United States v. Matlock, 415 U.S. 164 (1973)]. For third-party consent, joint access and control of the premises is required. In our problem, first and foremost, did the daughter have authority "in her own right" to allow the police to search her mother's bedroom? The answer is no. Although the child may have had the apparent authority to permit the police to search common areas of the home (e.g., living room or kitchen), the daughter clearly did not have authority to allow the police to search her mother's bedroom. Moreover, because the defendant locked the bedroom door, she did not assume the risk that her daughter would gain entry.

A college student was a committed vegan, eating only plant-derived foods, and the thought of all the animals slaughtered to provide meat for people nauseated her. One night, in order to draw attention to the slaughter of innocent animals, the student went to the butcher's section of the supermarket where she worked as produce manager and sprinkled a nausea-inducing chemical on all the meat, fowl, and fish products. Despite being careful, the student accidentally put too much powder in one package of hamburger. When the woman who bought the meat served it to her family, her husband, who was unusually susceptible, died as a result of ingesting the chemical. In the student's jurisdiction, murder is defined as "the unlawful killing of a human being with malice aforethought. Such malice may be express or implied. It is express when there is manifested a deliberate intention unlawfully to take away the life of another person. It is implied when no considerable provocation appears or when the circumstances attending the killing show an abandoned and malignant heart. All murder which is perpetrated by willful, deliberate, or premeditated killing or which is committed in the perpetration of or attempt to perpetrate arson, rape, robbery, or burglary is murder of the first degree. All other kinds of murders are of the second degree." If prosecuted for the criminal homicide of the woman's husband, of what crime should the student should be found guilty? (A) First-degree murder, because she willfully, deliberately, and with premeditation killed the victim. (B) First-degree murder, because the homicide occurred during perpetration of a felony. (C) Second-degree murder, because the circumstances show that she acted with an abandoned and malignant heart. (D) Involuntary manslaughter, because she did not intend to kill anyone

(D) Involuntary manslaughter, because she did not intend to kill anyone. Where there is no intent to kill, in order to constitute murder, a homicide must be either felony murder (which may be 1st-degree), 2nd-degree murder (intent to seriously injure/wanton and willful misconduct), or involuntary manslaughter (criminal negligence or misdemeanor manslaughter). In this case, the facts clearly indicate that the student had no intent to kill or seriously injure; she sought to make a political point by causing people who ate the affected foods to feel nauseated. Thus, the student could be guilty of felony murder, depraved-heart murder, or involuntary manslaughter. The statute given by the problem lists four felonies which, if a homicide occurs during the commission of them, could give rise to 1st-degree felony murder. Because the student committed none of these (arson, burglary, rape, or robbery), that basis for 1st-degree murder is eliminated. In choosing between depraved-heart murder & involuntary manslaughter, one must examine the degree of risk created by the conduct. Acting with a depraved heart is usually and more precisely defined as "wanton and willful misconduct"--conduct which the defendant knows will create a very high risk of death or serious bodily injury. Criminal negligence is usually regarded as equivalent to gross negligence--i.e., creating a high risk of death or injury--and if such conduct results in a homicide, the appropriate label is involuntary manslaughter. Under the circumstances of this question, the student's conduct appears to have created a high rather than a very high risk of injury, and is closer to gross negligence than to wanton and willful misconduct. For that reason, she is guilty of involuntary manslaughter.

An off-duty police officer came home early from work one day. He walked up to his bedroom and, upon opening the door, saw his wife and another man in bed together. Visibly upset, the husband grabbed his service revolver and shot the man to death. He then went to a local bar, had several drinks, returned home, and killed his wife. The husband is guilty for which of the following crimes? (A) Murder of both his wife and the other man. (B) Voluntary manslaughter of both his wife and the other man. (C) Murder of the other man, and voluntary manslaughter of his wife. (D) Murder of his wife and voluntary manslaughter of the other man.

(D) Murder of his wife and voluntary manslaughter of the other man. The husband killed the other man while in the heat of passion. There was adequate provocation, which will reduce this intentional killing from murder to manslaughter. However, the man did not initially kill his wife while under this provocation. Instead, he went to a bar, had several drinks, and then returned home to kill his wife. A reasonable person would have cooled off in this period. Therefore, the murder of the wife will not be reduced to manslaughter.

A teenage brother and sister had heard that a locally notorious immigrant, who had become extremely wealthy through a number of ethically questionable business ventures, did not trust American banks and kept all his money in a suitcase in his garage. One night, the teenagers decided to sneak into the man's garage and steal the suitcase full of cash. They jumped the fence surrounding his home and attempted to jimmy the garage door. They had just managed to enter the garage when his dog, alerted by the noises coming from the garage area, began barking furiously, causing the man to come out to investigate. In a panic, the brother grabbed a bicycle that was sitting in the garage and beckoned his sister to get on. They rode away as fast as they could, with the sister perched on the bike's handlebars, as the man chased after them yelling for help. As they turned a corner, however, they were met by a policeman who heard the commotion. Sensing that a crime had been committed, he ordered them to stop, but the brother kept pedaling. The policeman fired a shot at the tire of the bicycle in order to stop the fleeing teenagers, but the bullet struck and killed the sister. Can the brother be charged for his sister's death? (A) Yes, for felony murder, as they committed a burglary. (B) Yes, for involuntary manslaughter, because the brother was negligent in causing his sister's death. (C) No, because they had already left the scene of the felony. (D) No, because the deceased was a co-felon.

(D) No, because the deceased was a co-felon. Felony murder is an intentional or accidental killing proximately caused during the commission or attempted commission of a serious or inherently dangerous felony, such as burglary, arson, robbery, rape, and kidnapping (known by the mnemonic "BARRK"). For a defendant to be guilty of felony murder, the resulting death must be a foreseeable outgrowth of the defendant's actions. Courts have generally applied the foreseeability requirement very liberally, and for purposes of felony murder, most deaths are considered foreseeable. If a death occurs while the defendant is fleeing from the scene of the felony, he may still be guilty of felony murder. However, under the Redline limitation, a felon is not guilty of felony murder where the killing constitutes a justifiable homicide, such as where the police or the victim shoots one of the co-felons. In this case, inasmuch as a policeman shot the sister, his co-felon, the brother will not be criminally liable for his sister's death.

Police observed a customer leave the bar with a plastic cup in his hand and get into his car. The customer pulled into traffic and drove up onto the curb and then made a U-turn going the wrong way on a one way street. The officer pulled the car over and arrested the driver for DUI. The customer was handcuffed and seated in the back of the police car. The officer then conducted a search of the vehicle and found the plastic cup filled with beer under the driver's seat. The defendant filed a pre-trial motion contesting the legality of the search. Should the motion be granted? (A) Yes, because the defendant was already handcuffed and detained in the police car before the search took place. (B) Yes, because the officer did not have probable cause to look under the driver's seat. (C) No, because the search was conducted incident to a lawful arrest. (D) No, because the driver was a recent occupant of the vehicle and the officer had reason to believe that the car contained evidence of the crime for which the defendant was arrested.

(D) No, because the driver was a recent occupant of the vehicle and the officer had reason to believe that the car contained evidence of the crime for which the defendant was arrested. The recent occupant rule, formulated by SCOTUS in the Gant decision [Arizona v. Gant, 556 U.S. 332 (2009)], permits the officer to search a vehicle even if the defendant has been cuffed and detained in the police vehicle. In Gant, the court held "Police may search a vehicle incident to a recent occupant's arrest only if the arrestee is within reaching distance of the passenger compartment at the time of the search or it is reasonable to believe the vehicle contains evidence of the offense of arrest." What this means is that there are two situations in which the police may search the car of a recent occupant following his arrest: (1) if the arrestee is within reaching distance of the car; or (2) if the police believe the car contains evidence of the arrest. However, the search of the passenger compartment may only be conducted if they reasonably believe that evidence of the offense for which the defendant has been arrested is in the vehicle. In this case, the police observed the defendant leave a bar holding a plastic cup and drive in violation of traffic laws. Police certainly had a reasonable belief that evidence of drinking would be found in the passenger compartment, and therefore, the search was lawful.

A defendant was on trial for drug distribution. The arresting officer was called by the prosecution to describe in detail the events that led to the defendant's arrest. The officer described the night in question, where he was, and what he observed. He was asked to provide a list of what he confiscated from the defendant after he was arrested. The officer testified that the defendant has a large amount of money and many small packets of drugs in several pockets throughout his clothing. When asked specifically about the number of drugs packets, the officer was unable to recall. The officer was then shown his police report. The officer read over his report for several minutes. The prosecutor then asked if the officer was able to remember how many bags of drugs were taken from the defendant. The officer responded that he could not. At this point, the prosecutor directed the office to page three of his report and asked him to read the fifth line, which said, "Confiscated from the defendant's jacket were three purple packets with an off-white powdery substance, and confiscated from his right front pants pocket were 33 purple packets with an off-white powdery substance, that were placed on property receipt #43567." The defense attorney objected to the officer's testimony. How should the court rule? (A) Sustain the objection, because the officer lacks a present memory of the incident. (B) Sustain the objection, and require the report to be admitted because the jury is just as capable as reading the report as the officer. (C) Overrule the objection, because the officer's memory was refreshed. (D) Overrule the objection, because the report is admissible as a past recollection recorded.

(D) Overrule the objection, because the report is admissible as a past recollection recorded. The attempt to refresh the officer's memory was unsuccessful. Therefore, the prosecution needs to introduce the evidence by laying a foundation of the officer's report, and then have that portion of the report read into the record. Although the out-of-court statement is being offered for its truth, the past recollection recorded hearsay exception will provide a means of admissibility.

The defendant owned a 10-acre farm with an old well that was no longer used. His house was located on the middle of the property 1000 yards from the well. An anonymous tip led the police to believe that there was a gun that killed a murdered drug dealer that was hidden in the bottom of that well. The police searched the well on the defendant's property without a warrant and found the gun. The defendant is charged with murder. Which of the following is true? (A) The defendant had a Fourth Amendment right to a reasonable expectation of privacy because the gun was not visible from off the property. (B) The defendant had a Fourth Amendment right to a reasonable expectation of privacy on his entire plot of land, including the well where the police found the gun. (C) The defendant had no Fourth Amendment right to a reasonable expectation of privacy as to the gun because he had discarded the gun. (D) The defendant had no Fourth Amendment right to a reasonable expectation of privacy in the well, as it is outside the curtilage of the defendant's home.

(D) The defendant had no Fourth Amendment right to a reasonable expectation of privacy in the well, as it is outside the curtilage of the defendant's home. A defendant may assert his Fourth Amendment rights only when he has a reasonable expectation of privacy, which exists where the defendant has standing and where the objects to be seized are not "held out to the public." Any unoccupied or undeveloped area outside of the curtilage, or the living space directly around the home, is not protected by the 4th Amendment. Here, the police searched an area which was well away from his home. As such, the defendant did not have a reasonable expectation of privacy in this well, and this answer choice is correct.

The defendant owned a large plot of land in a rural area, and his house sat on the middle of the property. On the left side of the defendant's property sat an undeveloped wooded area, and on the right side of the defendant's property sat a guest house and a large garden. Recently, the defendant was involved in the robbery of a convenience store, which turned deadly when the store clerk was shot. The defendant took the gun that was used in the robbery and hid it in the woods on his property. After following a number of leads, police suspected that the defendant had played a part in the robbery and murder at the convenience store. They went to his house a number of times, but he refused to let them in or talk to them. The police searched the wooded area on the defendant's property without a warrant and found the gun. Which of the following is true? (A) The defendant had a Fourth Amendment right to a reasonable expectation of privacy because the gun was not visible from off the property. (B) The defendant had a Fourth Amendment right to a reasonable expectation of privacy on his entire plot of land, including the wooded area where the police found the gun. (C) The defendant had no Fourth Amendment right to a reasonable expectation of privacy as to the gun because he had discarded the gun. (D) The defendant had no Fourth Amendment right to a reasonable expectation of privacy in the wooded area on his property, as it is outside the curtilage of the defendant's home.

(D) The defendant had no Fourth Amendment right to a reasonable expectation of privacy in the wooded area on his property, as it is outside the curtilage of the defendant's home. A defendant may assert his 4th Amendment rights only when he has a reasonable expectation of privacy, which exists where the defendant has standing and where the objects to be seized are not "held out to the public." Any unoccupied or undeveloped area outside of the curtilage, or the living space directly around the home, is not protected by the 4th Amendment. Here, the police searched the undeveloped area on the left side of the defendant's property, which was away from his home. As such, the defendant did not have a reasonable expectation of privacy in this wooded area, and this answer choice is correct.

A sailor got into a fight with a marine while at a bar. The fight was quickly broken up by some other patrons. The sailor, however, remained angry. The sailor went back to the same bar the following night. While inside the bar, someone put a substance in the sailor's soda that made him extremely intoxicated. He soon left the bar and, 10 minutes later, ran into the marine on the sidewalk. The two men got into a verbal argument, during which the marine threatened to cause bodily injury to the sailor's girlfriend. The sailor became angry and pulled out a gun, shooting the marine to death. When the sailor woke up the next morning, the effect of the alcohol had mostly worn off. He had no recollection of what happened the night before. He did not remember having a gun, or using it to shoot the marine. This jurisdiction defines all murder as second-degree murder, but elevates intentional killings and those committed during an inherently dangerous felony to first-degree murder. Manslaughter is defined as at common law. Which of the following statements is most accurate? (A) The sailor is guilty of first-degree murder, because he intentionally killed the marine. (B) The sailor is guilty of second-degree murder, because, due to his intoxication, he could not formulate the specific intent to kill. (C) The defendant is guilty of voluntary manslaughter, because there was sufficient adequate provocation by the marine due to the threats made concerning the sailor's girlfriend. (D) The defendant is not guilty of homicide, because he lacked the state of mind required to commit the crime.

(D) The defendant is not guilty of homicide, because he lacked the state of mind required to commit the crime. Whereas voluntary intoxication may, in limited circumstances, prevent a defendant from forming the specific intent required for certain crimes, involuntary intoxication may be a complete defense to a variety of charges. Involuntary intoxication will excuse what would normally be criminal conduct if it: (1) prevents the defendant from understanding what he or she is doing; (2) causes the defendant to be unable to differentiate between right and wrong; (3) makes the defendant incapable of complying with the law; or (4) otherwise leaves the defendant lacking the culpable state of mind required for a conviction. Here, because the sailor became extremely intoxicated as a result of a substance that someone else put into his drink without his knowledge, his involuntary intoxication will operate as a defense to the homicide charge.

A man and his roommate got into an argument over a loan the man-made to the roommate. The roommate called the man a spoiled brat who always got his way. The man slapped the roommate across the face. The roommate then grabbed a carving knife from the kitchen counter and tried to stab the man. The man wrestled the knife away and stabbed the roommate in the chest, killing him. If the man is charged with homicide, which of the following statements is most accurate? (A) The man is guilty of murder, because deadly force was not justified. (B) The man is guilty of manslaughter, because deadly force was not justified. (C) The man is guilty of murder, because he was the initial aggressor. (D) The man is not guilty of any crime, because deadly force was justified.

(D) The man is not guilty of any crime, because deadly force was justified. The man provoked the attack by slapping the roommate across the face. However, the slap was nondeadly force. In a majority of jurisdictions, the roommate cannot respond to nondeadly force with deadly force, such as by using a knife. The roommate used excessive force in his response to the man's slap, and so the man can use deadly force to defend himself, and will not be responsible for criminal homicide under these circumstances.

Concerned by the number of large commercial properties being purchased by foreign nationals, a state legislature enacted a statute prohibiting the sale of commercial properties in excess of 20,000 square feet to aliens. A resident alien who has lived in the state for several years entered into a contract with his business partner to buy a commercial manufacturing business in the state. The factory was 30,000 square feet in size. The resident and the business partner decide to test the constitutionality of the state statute, and so sought a declaratory judgment in federal court. Which of the following statements regarding the declaratory judgment action is most accurate? (A) The business partner has the burden of showing that there is no rational basis for the statute's purposes regarding large commercial properties. (B) The court will find that the resident, but not the business partner, has standing. (C) The resident has the burden of showing that the state has no compelling interest in carrying out the statute's purposes regarding large commercial properties. (D) The state has the burden of showing a compelling state interest in carrying out the statute's purposes regarding large commercial properties.

(D) The state has the burden of showing a compelling state interest in carrying out the statute's purposes regarding large commercial properties. When a state statute discriminates against a suspect class such as aliens, the burden is on the state to show that its actions are necessary to achieve a compelling state interest. In this case, the state statute may violate the Equal Protection Clause by prohibiting aliens from acquiring large commercial properties. The federal court hearing the resident and the business partner's case will apply the strict scrutiny test to the statute to determine whether the law violates the constitutional guarantee of equal protection.

The plaintiff has held various public offices for many years, and is currently a state representative. After a recent scandal broke involving inappropriate text messaging with a minor, the plaintiff was convicted of felony obscenity charges, and was sentenced to time served and placed on probation. A week after the conviction was announced, the plaintiff was informed that, pursuant to a state statute, he was being immediately suspended from office without pay until such time as he had exhausted his appeals. Should his conviction become final, another statute authorized the attorney general to file suit to permanently remove the plaintiff from office, following a hearing on the matter and appellate review of the hearing decision. The plaintiff filed suit to prevent his suspension, claiming that the state statute was an unconstitutional bill of attainder and a violation of his due process rights. How should the court rule? (A) The plaintiff will prevail, because the statute violated his procedural due process rights. (B) The plaintiff will prevail, because the statute was an unconstitutional bill of attainder. (C) The state will prevail, because the statute provided for a judicial hearing. (D) The state will prevail, because the statute was not a bill of attainder.

(D) The state will prevail, because the statute was not a bill of attainder. Sections 9 and 10 in Article I of the U.S. Constitution bar both federal and state legislatures from passing bills of attainder. A bill of attainder is a legislative act that inflicts punishment without a judicial trial upon named individuals or an easily ascertainable group for past conduct. However, courts have held that laws suspending felons from public office or denying them the right to vote are not penal in nature, as their purpose is more to protect the integrity of the political process rather than to punish the criminal [Spooner v. West Baton Rouge Parish School Bd., 709 F. Supp. 705 (M.D. La. 1989); Trop v. Dulles, 356 U.S. 86 (1958)]. As such, the suspension statute in this state is not an unconstitutional bill of attainder.

A woman called 911, telling the operator that her husband had hit her, threatened her children, and was now breaking objects in the living room. Once the husband saw her calling 911, he fled, and the caller told the operator where he went. The husband was arrested and charged with assault. At trial, the woman did not testify. Instead, the prosecution called the 911 operator to play the recording of what the woman said in her call. The husband's attorney objected to the admission of the wife's call to the police. How should the trial court rule? (A) The statement is inadmissible because in criminal cases, all evidence presented against the defendant must be subject to cross-examination in front of the trier of fact. B) The statement is inadmissible because it was testimonial in nature. (C) The statement is admissible, as a statement made while under the stress of a startling event. (D) The statement is admissible because it was not testimonial in nature and qualifies as an excited utterance.

(D) The statement is admissible, because it was not testimonial in nature and qualifies as an excited utterance. This call would be nontestimonial in nature (it is describing current events to help police deal with an emergency), and so everything the wife said, including the identity of the attacker, would be admissible in court. Therefore, even if she does not show up in court, the prosecutor can play the tape in order to get a conviction.

A man pulled his truck onto the freeway and collided with a car. The driver of the car was seriously injured in the accident, and subsequently filed suit against the truck driver, alleging that he failed to yield as he entered the freeway from the entrance ramp. The truck driver, in turn, claimed that the driver of the car was speeding and had failed to make any attempt to avoid the accident even though the two lanes to the left of the entrance ramp were wide open. The truck driver's attorney called the truck driver to testify that when he approached the car immediately following the accident, the driver of the car stated over and over, "I should have slowed down or moved over." The driver of the car objected. Should the truck driver be allowed to testify as to the driver of the car's statement? (A) No, because it is inadmissible hearsay. (B) No, because the truck driver is an interested party and there is no corroboration of the statement. (C) Yes, because it shows awareness by the driver of the car that he was speeding. (D) Yes, because it is a statement by an opposing party.

(D) Yes, because it is a statement by an opposing party. Under FRE 801, a statement by an opposing party that is offered against that party is non-hearsay. Here, the car driver's statement constitutes an opposing party's statement, and will therefore be admissible against him at trial

A CEO was addressing a large group of shareholders at an annual meeting in which his opening remarks discussed the health of the business, some recent acquisitions, and his personal health. "It was a long trip back," he said, "but I am feeling great, ready to take on the world." This news immediately hit the media and stock prices jumped 2%, as there was speculation that the CEO was suffering from severe depression. The following morning, the CEO was found dead in his hotel room. After his death, the CEO's wife attempted to collect on his life insurance policy, but her claim was denied due to speculation that the CEO committed suicide. At trial, the wife sought to use the CEO's statement to the shareholders that he was feeling great and ready to take on the world. The insurance company objected. Should the trial judge admit the testimony? (A) No, because there was speculation that he suffered from depression. (B) No, because it was not made to a doctor. (C) Yes, because it is relevant to the CEO's medical condition. (D) Yes, because it is a statement of the CEO's then-existing physical condition.

(D) Yes, because it is a statement of the CEO's then-existing physical condition. FRE 803(3) allows for admission of an out-of-court statement describing the declarant's then-existing mental, emotional, or physical condition. This includes a statement of the declarant's then-existing state of mind (such as motive, intent, or plan) or emotional, sensory, or physical condition (such as mental feeling, pain, or bodily health), but not including a statement of memory or belief to prove the fact remembered or believed unless it relates to the validity or terms of the declarant's will. Here, the proponent (the CEO's wife) is attempting to show that her husband was not likely to have killed himself based on his comment the day before about how he was feeling. Because she wants to offer this statement for its truth and it discusses how the CEO was currently feeling, it falls within this exception.

A man was arrested for sabotaging an electrical plant and causing a blackout in the city. The police suspected that there was another electrical plant that was also a target for sabotage but could not make the man admit his guilt or help the police in any way. A rookie police officer feared the harm to the community if there was another blackout and searched the man's home and office without a warrant. He found a computer file with information on the next target and how to stop the sabotage. Five minutes later, other police officers who did not know of the rookie police officer's action and pursuant to a search warrant were conducting a systematic search of the man's property beginning with his house and then his office. Will the computer file be admissible as evidence in the man's subsequent criminal trial for sabotaging the electrical plant? (A) No, because it is "fruit of the poisonous tree" and therefore tainted. (B) No, because the officers who had the search warrant were not the ones who discovered the document. (C) Yes, because there was an urgent need to search the man's office in order to neutralize the threat to the electrical plant as quickly as possible. (D) Yes, because officers would have inevitably discovered the evidence when they searched the suspect's office pursuant to the search warrant.

(D) Yes, because officers would have inevitably discovered the evidence when they searched the suspect's office pursuant to the search warrant. In general, the evidence that the officer illegally obtained in violation of the defendant's Fourth Amendment rights would be excluded under the exclusionary rule. However, there are several exceptions to the exclusionary rule. One such exception exists where the evidence in question would have been inevitably discovered regardless of the illegal discovery. In this case, police had already obtained a search warrant for the man's home and office, and they were in the process of executing that search warrant. They simply hadn't gotten to the office yet. Therefore, since they would have inevitably discovered the computer document using the search warrant, it is admissible, even though it was illegally discovered by the rookie police officer. Thus, this answer choice is correct.

After a nine-year-old girl was kidnapped while walking home from school, police and community members began to search the neighborhood. While the search for the girl was ongoing, police also launched an extensive investigation of the kidnapping in an effort to identify a suspect. After several days, police identified a neighbor as a suspect in her kidnapping and brought him into the station for questioning. A rookie police officer had heard that after 48 hours the chance of rescuing a kidnapped child alive decreased dramatically. Since more than 48 hours had passed since the girl was kidnapped, the officer became desperate. He asked around until he learned the suspect's identity and took it upon himself to break into the suspect's office and search the files on his computer for evidence. The officer found a document on the computer which appeared to be a detailed description of the kidnapping, so he printed the document and returned to police headquarters. At the same time, pursuant to a search warrant, other officers on the case were conducting a systematic search of the suspect's property beginning with his house and then his office. Will the computer file be admissible as evidence in the suspect's subsequent criminal trial for the kidnapping? (A) No, because it is "fruit of the poisonous tree" and therefore tainted. (B) No, because the officers who had the search warrant were not the ones who discovered the document. (C) Yes, because there was an urgent need to search the suspect's office in order to locate the girl as quickly as possible. (D) Yes, because officers would have inevitably discovered the evidence when they searched the suspect's office pursuant to the search warrant.

(D) Yes, because officers would have inevitably discovered the evidence when they searched the suspect's office pursuant to the search warrant. In general, the evidence that the rookie officer illegally obtained in violation of the defendant's Fourth Amendment rights would be excluded under the exclusionary rule. However, there are several exceptions to the exclusionary rule. One such exception exists where the evidence in question would have been inevitably discovered regardless of the illegal discovery. In this case, police had already obtained a search warrant for the suspect's home and office, and they were in the process of executing that search warrant. They simply hadn't gotten to the office yet. Therefore, since they would have inevitably discovered the computer document using the search warrant, it is admissible, even though it was illegally discovered by the rookie police officer. Thus, this answer choice is correct.

A man crashed into a woman's car. Shortly after the accident, a police officer arrived at the scene, and the woman told the officer that the man had run the red light and swerved into her car. The police officer wrote her statement in the "Accident Report Form," which he was required to complete after every accident. The woman sued for personal injuries sustained in the accident. On direct examination, her attorney called her to testify and asked her to recall the events of the night. Is the woman's testimony admissible? (A) No, because a party is precluded from introducing their own out-of-court statements at a subsequent trial. (B) No, because it is hearsay not within any exception. (C) Yes, because the officer wrote down the statement as part of his official duties. (D) Yes, because the woman has firsthand knowledge of the events of the night in question.

(D) Yes, because the woman has firsthand knowledge of the events of the night in question. The woman is not being asked to recount her out-of-court statement, but rather, to testify as to what she remembered seeing on the night in question. As long as the witness has firsthand knowledge, she is a proper witness to the events, and can testify as to what she saw. Therefore, there is no hearsay problem.

A city with a majority nonwhite population has historically had a nonwhite police commissioner. After a recent election, the new city mayor appointed the first white police commissioner in decades. Two months into the job, the new commissioner instated a "qualification exam" requirement for all current police officers and those seeking to become officers. The stated purpose of this exam was to "ensure that only the most qualified people are upholding the laws of this city and state." Current officers who failed the exam would have their employment terminated, and new applicants would need to pass the exam before being considered to join the police academy. For those who failed the exam, an appeals process was also put in place, allowing the applicant to argue that they were qualified for the position despite failing the written exam. In the wake of the new requirements, 80% of the nonwhite police officers failed the exam and were fired, while only 20% of the white officers failed. Of those who appealed, over 75% of white officers had their appeals granted, while only 10% of nonwhite officers had theirs granted. After six months, it was found that white applicants passed the test 68% of the time, while only 12% of nonwhite applicants passed. Half of the white applicants who appealed were admitted to the police academy, while no nonwhite applicant's appeal was granted. Is the city's qualification exam requirement constitutional? (A) No, because tests that disadvantage racial minorities receive strict scrutiny and are presumptively unconstitutional. (B) No, because the test has a disparate impact on the success of racial minority candidates. (C) Yes, because a state's police powers includes the absolute right to prescribe qualifications for city police officers. (D) Yes, because there is no evidence of intentional race discrimination in the administration of the test.

(D) Yes, because there is no evidence of intentional race discrimination in the administration of the test. In Washington v. Davis [426 U.S. 229 (1976)], the Supreme Court held that race-neutral qualifications for state offices did not trigger heightened (i.e., intermediate or strict) scrutiny in the absence of evidence of a discriminatory purpose. In Davis and other cases, the Court has made clear that a disproportionate impact on distinct racial groups, standing alone, is insufficient to show intentional discrimination and trigger heightened scrutiny. However, when a facially neutral law involves discretionary decisions, and those decisions are made in a discriminatory manner, the Supreme Court has found that there is purposeful discrimination in the application of the law [Yick Wo v. Hopkins, 118 U.S. 356 (1886)].

A customer has banked with the same area bank for the last 10 years. On several occasions, the bank merged with or was bought out by a larger national bank, but in each instance, the customer was able to iron out the ensuing logistical difficulties and maintain his fee-free banking services, in large part because of a very large account he has at the bank. Now, the bank has merged again. Two days after the merger, the customer receives several statements documenting substantial fees. The customer is enraged. He contacts the newly appointed bank manager, who indicates that that she is not willing to help the customer oppose the fees. The customer requests the name of her supervisor, and the manager tells him that in fact, she herself is the regional manager and oversees the entire state. The manager then hangs up on the customer. Infuriated, the customer walks over to the bank, enters, and walks straight to the manager's office. He fires a single shot at the woman standing in the office, killing her. The woman in the office was not the manager but another customer waiting for the manager to return from the vault. The customer is facing charges for the shooting death. Of which of the following crimes is the defendant guilty? (A) First-degree murder. (B) Second-degree murder. (C) Voluntary manslaughter. (D) Involuntary manslaughter

A) First-degree murder. First-degree murder includes intent-to-kill murder committed with premeditation and deliberation, felony murder, and, in some jurisdictions, murder accomplished by lying in wait, poison, or torture. If a murderer engages in any reflection or premeditation prior to the homicide, even if the reflection is cursory and brief, he may be guilty of first-degree murder. Here, although the person killed was not the defendant's intended victim, the defendant's factual mistake as to the identity of the person in the manager's office does not relieve him of responsibility for his purposeful act of killing. Rather, the defendant's intent to kill the manager is transferred to the person killed. Because the defendant acted with premeditation and deliberation, first-degree murder is the crime of which the defendant is guilty.

A man lived next door to a woman. Late one night, the man overheard the woman scream "Ellen, put down the knife, please don't do it!" The next day, the woman was found dead in her bedroom. Ellen was arrested and charged with murdering the woman. At trial, the man proposed to testify to the woman's statement. Upon objection by the defendant's counsel, how should the court rule on the man's testimony regarding the woman's statement? (A) Admissible as a dying declaration, if the jury determines that the woman believed that her death was imminent. (B) Admissible as a dying declaration, if the judge, by a preponderance of the evidence, determines that the woman believed that her death was imminent. (C) Inadmissible, because the probative value is substantially outweighed by the danger of unfair prejudice. (D) Inadmissible, as hearsay not within any recognized exception.

B) Admissible as a dying declaration, if the judge, by a preponderance of the evidence, determines that the woman believed that her death was imminent. Under FRE 104(a), "Preliminary questions concerning the qualification of a person to be a witness, the existence of a privilege, or the admissibility of evidence shall be determined by the court." The judge is not bound by the rules of evidence at this point, except those with respect to privileges. To the extent that these inquiries are factual, the judge acts as a trier of fact. The judge's decision is final and is not subject to a contrary determination by the jury. On the other hand, the jury is to determine how much, if any, probative value or "weight" to accord to the admitted evidence, as well as to decide issues of credibility (i.e., whether to believe, wholly or in part, the witness's testimony). The judge not only decides factual issues, he also determines the applicability of any technical evidentiary rules. He decides, for example, if a dying declarant had a sense of impending death; if an entry was made promptly in the regular course of business; if there was the necessary state of excitement to qualify a declaration as an excited utterance; if a witness is unavailable; or if an original document is unavailable so as to justify the admission of a copy under the best evidence rule. Finally, note that preliminary questions of fact are ordinarily resolved in both criminal and civil trials by using a preponderance of the evidence standard. The witness's testimony as to the victim's statement will be admissible as a dying declaration if the judge, by a preponderance of the evidence, determines the victim believed her death was imminent. Thus, this answer choice is correct.


Conjuntos de estudio relacionados

Ch 3 Into, Conc., Homework Practice: The accounting cycle: end of the period

View Set

FIN 320-51 Ch. 5 Quiz, FIN 320-51 Ch. 6 Quiz, FIN 320-51 Ch. 7 Quiz, FIN 320-51 Ch 8 Quiz, FIN 320-51 Ch 9 Quiz

View Set

Excel PivotTables and PivotCharts

View Set